Whole Anatomy
Whole Anatomy
Whole Anatomy
85. A doctor while performing bronchoscopy, he 90. During a motorcycle accident an 18-year old
passes the bronchoscope down the trachea, a male landed on the right lateral side of his rib
cartilaginous structure is observed separating cage with his upper limb abducted. On
the trachea into right and left main bronchus examination he was found to have winging of
is called as: right scapula. Which of the following nerve is
A. Tracheal ring likely to be damaged in this patient:
B. Cricoid cartilage A. Long thoracic
C. Carina B. Vagus
D. Costal cartilage C. Accessory
E. Copula D. Phrenic
E. Lateral pectoral
86. In a female typist having pregnancy of 08-
months with high blood pressure, there is
wasting of thenar muscle. Which nerve is
most likely to be affected in this lady:
A. Suprascapular nerve
91. A newborn was brought to the physician with 96. A 12-years old boy with complain of chest
history of difficult labour (child birth), which pain and dyspnea. On auscultation abnormal
causes injury to upper trunk of brachial heart sound / murmur was found. For
plexus. Which of the following is most likely auscultation of mitral valve where doctor
to be seen in baby: should put a stethoscope:
A. Sensation loss on medial side of arm A. In left 2nd intercostal space
B. Arm lies in lateral rotation B. In left 3rd intercostal space
C. Paralysis of rhomboid major and minor C. In right 2nd intercostal space
D. No lateral rotation and abduction D. Mid sternal region
E. Abduction lost beyond 90 ͦ E. In left fifth intercostal space
92. A young anatomist while doing dissection is 97. A 25-years old lady having history of chronic
keenly interested in localizing neurovascular liver disease, complain of severe bleeding
bundle of thoracic wall, where he can exactly from vomiting after taking food. Which one
find this structure: of the following is the probable cause of
A. In the subcutaneous tissue of thoracic wall bleeding:
B. Between outer and middle muscular layer A. Rupture of esophageal veins in upper 1/3rd
C. Between middle and inner most muscular of esophagus
layer B. Rupture of esophageal veins in lower 1/3rd
D. Under the transversus thoracic muscle of esophagus
E. Under endothoracic fascia C. Rupture of lower esophageal sphincter
D. Rupture of azygous vein draining esophagus
93. In a patient of 60-years old a tumor of E. Rupture of esophageal veins in middle
posterior mediastinum which of the following
structure may be compressed: 98. A 50-years old patient brought to emergency
A. Ascending aorta department with complain of chest pain and
B. Trachea headache. His CT scan shows tumor located
C. Descending aorta just superior to the root of right lung. Which
D. Arch of azygous vein of the following vein most likely blocked by
E. Arch of aorta this tumor:
A. Right subclavian vein
94. A 05 days old baby boy get cyanosed on B. Right brachiocephalic vein
crying was brought to emergency department C. Arch of azygous vein
and diagnosed as a case of patent foramen D. Accessory hemiazygous vein
ovale. Embryologically this defect is E. Hemiazygous vein
associated with which of the following
structure: 99. A 30-years old patient brought to ER by his
A. Septum primum relatives, with history of severe chest injury
B. Septum secondum during car accident. Chest X-ray revealed
C. Septum spurium pleural effusion. Thoracocentesis was
D. Intra-atrial defect performed that reveals the presence of
E. Intra-ventricular defect chylous fluid, suggesting a rupture of a
thoracic duct. Which of the following
95. During development of body cavities mediastina contain the thoracic duct:
pleuropericardial folds contain which of the A. Superior and anterior mediastinum
following structure / structures: B. Superior and posterior mediastinum
A. Common cardinal veins C. Middle and superior mediastinum
B. Common cardinal vein and phrenic nerve D. Anterior and posterior mediastinum
C. Posterior cardinal vein and phrenic nerve E. Middle and anterior mediastinum
D. Anterior and posterior cardinal veins
E. Phrenic nerve and cardiac plexus
100. A young boy has got insect bite on C. Angle
dorsum of hand. He feels pain, itching and D. Shaft
temperature at one of bite. Which of the E. Head
following nerve ending will defect these
sensation: 106. After an incised wound on the front of the
A. Meissner’s corpuscles wrist, the patient is unable to perform
B. Pacinian corpuscles abduction and adduction of his fingers and
C. Riffini corpuscles adduction of his thumb. Sensations of the skin
D. Free nerve endings of the palm were intact. The most likely nerve
E. None of above injured is the:
A. Deep branch of the ulnar nerve
101. Which of the following type of epithelium B. Dorsal branch of the ulnar nerve
is found in skin: C. Main trunk of the median nerve
A. Simple squamous epithelium D. Recurrent branch of the median nerve
B. Pseudostratified squamous epithelium E. Superficial branch of the ulnar nerve
C. Stratified squamous epithelium
D. Stratified columnar epithelium 107. After a right axilla stab wound, the
E. None of above patient felt difficulty in abduction and flexion
of the right arm above 45 degrees. The
102. A 75-years old patient has been suffering scapula on the same side is prominent. The
from lung cancer located near the cardiac most likely injured nerve is the:
notch. Which of the following lobes is most A. Axillary
likely to be excised: B. Dorsal scapular
A. Superior lobe of the right lung C. Long thoracic
B. Middle lobe of the right lung D. Suprascapular
C. Inferior lobe of the right lung E. Thoracodorsal
D. Superior lobe of the left lung
E. Inferior lobe of the left lung 108. During the procedure of taking a blood
sample from the median cubital vein. Which
103. A damaged heart muscle resulting from one of the following nerve is most likely to be
occlusion of the circumflex branch of the left pierced:
coronary artery would most likely be found A. Medial cutaneous nerve of the arm
in the: B. Medial cutaneous nerve of the forearm
A. Apex C. Median nerve
B. Right atrium and right ventricle D. Musculocutaneous nerve
C. Left atrium and left ventricle E. Radial nerve
D. Right and left ventricle
E. Right ventricle and the interventricular 109. A patient presents with abscess in the first
septum web space of the hand. Which of the following
lymph nodes will be first affected by this
104. A young boy has blood pressure 160/110 infection:
mm Hg in the right arm and 140/90 mm Hg A. The pectoral
in the left arm with weak femoral pulse. The B. The epitrochlear
most likely site of lesion is: C. The supraclavicular
A. Ductal coarctation D. The infraclavicular
B. Narrowing abdominal aorta E. The lateral
C. Obliteration of left axillary artery
D. Post ductal coarctation of aorta 110. A patient is unable to abduct his right
E. Pre ductal coarctation of aorta arm and supinate his forearm. Which of the
following group of nerves is damaged:
105. Which part of rib is most likely to be A. The whole brachial plexus
fracture / affected: B. The axillary nerve
A. Neck C. The musculocutaneous nerve
B. Tubercle D. The upper portion of the brachial plexus
E. The lower portion of the brachial plexus 116. A 45-year old woman has a severe
asthmatic exacerbation and requires an
111. An 18-years old boy involved in an arterial blood gas specimen for management.
automobile accident present with arm that If you are planning to draw the sample from
cannot abduct. His paralysis is caused by the brachial artery, where should you insert
damage to which of the following nerves: the needle in the cubital fossa:
A. Suprascapular and axillary A. Superficial to the bicipital aponeurosis
B. Thoracodorsal and upper subscapular B. Superficial to the median cubital vein
C. Axillary and musculocutaneus C. Medial to the tendon of the biceps
D. Radial and lower subscapular D. Lateral to the tendon of the biceps
E. Suprascapular and dorsal scapular E. Medial to the median nerve
112. A baby born by forceps delivery with a 117. If the bone forming capacity of the
trauma on an arm cannot abduct the arm and mesenchyme is restricted to paraxial
supinate the forearm along with weakness of mesoderm, then which of the following bone
flexion of the elbow joint. Which of the only develops:
following nerve is most likely damaged: A. Bone of pectoral girdle
A. The musculocutaneous nerve B. Bones of limbs
B. The median nerve C. Bones of pelvis
C. C5, C6 and C7 D. Sternum
D. The ulnar nerve E. Bony part of rib
E. The radial nerve
118. Which of the following possesses a
113. A patient comes in complaining that she distinct internal elastic lamina:
cannot flex her proximal interphalangeal A. Capillary
joints. Which of the following muscles B. Metarteriole
appears to be paralyzed on further C. Arteriole
examination of her finger: D. Muscular artery
A. . E. Vein
B. .
C. . 119. After maturation in thymus T-
D. . lymphocyte migrate preferentially to which of
E. . the following sites:
A. Paracortex of the lymph nodes
114. A 23-year old accountant trips over a B. Cortical lymphoid nodules of lymph nodes
briefcase and falls onto his outstretched hand, C. Hilus of the lymph nodes
a carpal bone fracture is suspected. Which of D. Lymphoid nodules of tonsil
the following bones is most likely fractured: E. Lymphoid nodules of the spleen
A. Lunate
B. Scaphoid 120. Hand and foot plate are formed in which
C. Hamate week of development:
D. Capitate A. 4th week
E. Pisiform B. 5th week
C. 6th week
115. A 24-year old male slips on a banana peel D. 7th week
and falls onto his outstretched hand. Which E. 8th week
of the following structures transmits the force
from the radius to the ulna: 121. A histological section under microscope
A. Wrist joint shows well defined simple squamous
B. Elbow joint epithelium in tunica intima, walls are less
C. The interosseous membrane muscular having elastic tissue. Which type of
D. Ulnar collateral ligament vessel it is:
E. Radial collateral ligament A. Conducting artery
B. Distributing artery
C. Arteriole
D. Venule 126. During lymph nodes biopsy in the
E. Muscular artery posterior mediastinum, the thoracic duct is
accidently cut. The resulting accumulation of
122. A newborn presented with omphalocele in lymph in the pleural cavity is referred to as:
pediatric surgery ward duty doctor while A. Pleurisy
discussing the case told that this occurs B. Chylothorax
because of failure of physiological herniation C. Pyothorax
to return back in abdominal cavity, which is D. Hemothorax
normally seen in which week of development: E. Lymphedema
A. During 1st to 2nd week
B. During 3rd to 4th week 127. A pregnant lady referred for ultrasound
C. During 6th to 10th week for fetal wellbeing reports of scan show non-
D. During 4th to 5th week spiral course of conotruncal septum. Which
E. During 6th to 8th week congenital defect you may see in the
newborn:
123. A sick person lying supine in bed A. Tetralogy of fallot
aspirates (breathes in) some fluid into her B. Patent truncus arteriosus
lungs while swallowing. It would most likely C. Transposition of great arteries
end up in which of the following D. Coarctation of aorta
bronchopulmonary segments: E. Foramen ovale
A. Anterior segmental bronchus of right
superior lobe 128. Which part of vascular system is
B. Medial segmental bronchus of right middle responsible for regulation of blood pressure:
lobe A. Elastic artery
C. Superior segmental bronchus of right B. Muscular artery
inferior lobe C. Arteriole
D. Medial basal segmental bronchus of left D. Venules
inferior lobe E. Lymphatics
E. Inferior segmental bronchus of lingular lobe
129. Most of the drainage of the thoracic body
124. While performing trans-esophageal wall reaches the superior vena cava via the
echocardiography on a patient, the posterior azygous vein. A notable exception is the left
wall of the esophagus immediately behind the superior intercostal vein which normally
left antrum was punctured from within, drains into the:
doctors were also very concerned about A. Left bronchiocephalic vein
possible damage to a thin walled vessel just B. Left bronchial vein
behind the esophagus and between the C. Left pulmonary vein
azygous vein and aorta i.e.: D. Left subclavian vein
A. Hemiazygous vein E. Superior vena cava
B. Left bronchial vein
C. Left pulmonary vein 130. A tumor of the posterior mediastinum is
D. Superior vena cava most likely to compress which of the
E. Thoracic duct following structures:
A. Arch of aorta
125. An 8-year old boy is found to have a mid- B. Esophagus
line tumor of the thymus gland that is C. Inferior vena cava
impinging posteriorly on a blood vessel. The D. Pulmonary truck
affected vessel is most likely the: E. Trachea
A. Left bronchiocephalic vein
B. Left pulmonary vein
C. Left bronchial vein
D. Right pulmonary artery
E. Right superior intercostal vein
131. Which of the following layer provides a E. Superior vena cava
natural cleavage plane for surgical separation
of the costal pleura from the thoracic wall: 136. In absence of inferior vena cava blood
A. Deep fascia from caudal part of body reaches the heart by
B. Endothoracic fascia the way of azygous vein. Name the
C. Parietal pleura embryological source of origin of azygous
D. Visceral pleura vein:
E. Transversus thoracic muscle fascia A. Posterior cardial
B. Superior cardial
132. The ductus arteriosus sometimes remains C. Supra and sub cardial
open after birth requiring surgical closure. D. Posterior cardial and supra cardial
When placing a clamp on the ductus, care E. Sacro cardial
must be taken to avoid injury to what
important structure immediately dorsal to it: 137. A 39 years old mountaineer presents to
A. Accessory hemiazygous vein the emergency department after sustaining a
B. Left internal thoracic artery laceration of anatomical snuff box with
C. Left phrenic nerve profuse bleeding in a rock climbing accident.
D. Left recurrent laryngeal nerve Which of the following structure is most
E. Thoracic duct likely to be injured:
A. Superficial palmar arch
133. A 68-year old male who has large B. Opponens pollicis
amounts of fluid in the left pleural cavity. C. Radial artery
When you examine him in sitting position, D. Recurrent branch of median nerve
where would this fluid tends to accumulate: E. Deep branch of radial nerve
A. Costo-diaphragmatic recess
B. Costo-mediastinal recess 138. Partial / complete absence of skeletal
C. Cupola muscle of upper limb is the defect of which
D. Hilar reflection germ layer:
E. Middle mediastinum A. Ectoderm
B. Endoderm
134. During a heart transplant procedure, the C. Paraxial mesoderm
surgeon inserted his left index finger through D. Splanchnic mesoderm
the transverse pericardial sinus and then E. Parietal mesoderm
pulled forward on the two large vessels lying
ventral to his finger. Which vessels were 139. A person receives a laceration along the
these: anterior border of the trapezius muscle in the
A. Pulmonary trunk and tracheocephalic trunk neck and subsequently the point of his
B. Pulmonary trunk and aorta shoulder (scapula) sags and he has some
C. Pulmonary trunk and superior vena cava difficulty in fully abducting his arm. Which
D. Superior vena cava and right pulmonary nerve appears to have been severed:
artery A. Accessory (Cranial Nerve XI)
E. Superior vena cava and aorta B. Axillary
C. Greater occipital
135. A patient involved in an automobile D. Suprascapular
accident presents with a sharp object E. Median
puncture of the middle of the sternum at
about the level of the 4th or 5th costal cartilage. 140. In a section of lung tissue consisting
If the object also penetrated pericardium and predominantly of alveoli, a tubule of about 2
heart wall, which structure would most likely mm in diameter that contains smooth muscle
be damaged: and cartilage in its wall. The tubule is which
A. Left atrium one of the following:
B. Left ventricle A. Alveolar duct
C. Right atrium B. Alveolar sac
D. Right ventricle C. Bronchiole
D. Bronchus D. Weakness in extension of thumb
E. Trachea E. Weakness of the first and second lumbricals
141. The entire lymphatic region of the spleen 145. After automobile accident examination
is called as which one of the following: revealed that the medial border and inferior
A. Malpighian corpuscle angle of the left scapula became unusually
B. Trabeculae prominent when the arm was carried forward
C. White pulp in the sagittal plane, especially if the patient
D. Red pulp pushed with outstretched arm against heavy
E. Cords of billroth resistance e.g. a wall. What muscle must have
been denervated:
142. During a biking accident, a rider fell and A. Levator scapulae
landed with the handle bar of her bike forced B. Pectoralis major
upward into her right axilla. Subsequently C. Rhomboideus major
while swimming she found that her right arm D. Serratus anterior
tired so badly during the swimming. During E. Subscapularis
examination it was found that movements
involving adduction, medial rotation and 146. During the planning of therapeutic
extension of her arm were particularly weak intervention for a 54-year old female patient
and affected her swimming stroke. The nerve with cancer of the right breast. A 3rd year
injured was the: medical student would need to first consider
A. Accessory where most of the cancer cells would
B. Dorsal scapular metastasize:
C. Lateral pectoral A. Abdominal wall
D. Medial pectoral B. Anterior mediastinum
E. Thoracodorsal C. Axillary lymph nodes
D. Opposite breast
143. A 15-year old boy is brought to the E. Parasternal lymph nodes
emergency department because of injuries to
his head and right shoulder that he sustained 147. After a severe injury to the left anterior
while riding his bicycle. The boy is holding shoulder region, a young field hockey player
his right arm with the palm facing was told by physician that she had a muscle
posteriorly. He is unable to abduct the arm tear that resulted directly from the
and there is diminished sensation over the superolateral distraction of a fractured
lateral aspect of the arm. An injury of which coracoid process. Which muscle was torn:
of the following is most likely responsible for A. Deltoid
this patient’s symptoms: B. Pectoralis major
A. Axillary nerve C. Pectoralis minor
B. Lateral cord of brachial plexus D. Serratus anterior
C. Musculocutaneous nerve E. Subclavius
D. Suprascapular nerve
E. Upper trunk of the brachial plexus 148. Pen test in the hand is performed to assess
the neuromuscular status of which muscle:
144. A patient comes to the emergency A. Opponens pollicis
department after driving a knife into her B. Flexor pollicis brevis
palm while trying to slice bread. On follow up C. Abductor pollicis brevis
examination several weeks later the thenar D. 1st palmar interossei
eminence is observed to be flattened. Which E. 1st dorsal interossei
of the following additional findings is most
likely to be seen in this patient:
A. Inability to spread the thumb away from the
fingers in same plane
B. Weakness in pronation
C. Weakness in adduction of thumb
149. A girl playing softball cuts the palm of 155. The wrist joint is a:
her hand as she scoops up a piece of glass A. Synovial hinge joint
along with the ball. If the only nerve damaged B. Synovial pivot joint
is the recurrent branch of the median nerve, C. Synovial ball and socket joint
she would lose what movement of the thumb: D. Synovial ellipsoid joint
A. Abduction E. Synovial saddle joint
B. Adduction
C. Flexion of the distal phalanx 156. The structure that passes superficial to
D. Opposition the flexor retinacula is:
E. Extension of distal phalanx A. Median nerve
B. Cephalic vein
150. A worker doing repetitive lifting develops C. Abductor pollicis longus
an inflammation in the tendon of origin of the D. Ulnar artery
extensor muscles, commonly called “tennis E. Flexor digitorum superficialis tendon
elbow”. The focal point of pain would most
likely be near which palpable bony 157. The radial artery on entering the palms
landmark: by leaving the dorsum of the hand passes
A. Coronoid process of ulna between two heads of the:
B. Lateral epicondyle of humerus A. 2nd dorsal interosseous muscle
C. Lateral supracondylar ridge of humerus B. 3rd dorsal interosseous muscle
D. Medial epicondyle of humerus C. 1st dorsal interosseous muscle
E. Medial supracondylar ridge of humerus D. 1st palmar interosseous muscle
E. 2nd palmar interosseous muscle
151. Suprascapular nerve originates from:
A. Roots of brachial plexus 158. The lymph from the medial side of the
B. Lower cord hand drains into the:
C. Lower truck A. Interclavicular nodes
D. Upper trunk B. Supratrochlear nodes
E. Medial cord C. Pectoral nodes
D. Subscapular nodes
152. Which of the following nerve is damaged E. Infratrochlear nodes
if a person is unable to oppose his thumb to
little finger: 159. The structure related on the right side of
A. Ulnar nerve the trachea is:
B. Median nerve A. Arch of aorta
C. Radial nerve B. Esophagus
D. Anterior interosseous nerve C. Azygous vein
E. Posterior interosseous nerve D. Subclavian arteries
E. Thymus
153. The nerve supplying the deltoid muscle
also supplies: 160. The quadrangular space is bounded
A. Supra spinatus medially by the:
B. Infra spinatus A. Long head of the triceps
C. Teres major B. Surgical neck of the humerus
D. Teres minor C. Subscapularis muscle
E. Subscapularis D. Capsule of the shoulder joint
E. Teres major muscle
154. The serratus anterior muscle is inserted
into: 161. A 02-year old boy was playing with his
A. Lateral lip of bicipital groove of humerus toy car. When his mother noticed that a small
B. Clavicle metal nut was missing from the car. Two days
C. Corocoid process of scapula later the child developed cough and difficulty
D. Upper six costal cartilages in breathing. On bronchoscopy it was found
E. Medial border and inferior angle of scapula that the nut has been aspirated by him.
Which is the common site of respiratory tract E. Vagus nerve
for foreign body impaction:
A. Larynx 167. The anterior and posterior circumflex
B. Trachea humeral arteries form an anastomosing circle
C. Left lung around the:
D. Right lung A. Anatomical neck of humerus
E. Left superior lobar bronchus B. Head of the humerus
C. Surgical neck of humerus
162. A 19-year old man presents with pain on D. Greater tuberosity of humerus
movement of wrist. There is history of fall on E. Lesser tuberosity of humerus
outstretched hands few days back.
Examination reveals tenderness in area of 168. The aortic opening in the diaphragm lies
snuff box. X-ray finding shows a fracture of the level of which of the following thoracic
the following carpal bone: vertebra:
A. Pisiform A. 10th
B. Capitate B. 12th
C. Scaphoid C. 8th
D. Lunate D. 9th
E. Hamate E. 7th
163. The cardiothoracic surgeon decided to do 169. The intercostal nerves and vessels in the
heart surgery. The ideal route for him to open costal groove are arranged from above
the heart is through anterior wall of thoracic downwards as:
cage. The anterior wall of the thoracic cage is A. Nerve, artery and vein
formed by the: B. Vein, artery and nerve
A. Vertebral column C. Artery and nerve
B. Ribs D. Vein, nerve and artery
C. Intercostal spaces E. Artery, nerve and vein
D. Sternum and costal cartilages
E. Diaphragm 170. Superior border of the heart is formed
by:
164. After mediastenotomy of superior A. Left ventricle and right auricle
mediastinum the patient noticed a change in B. Right and left atria
his voice. What could be the possible cause of C. Left ventricle and left atria
this condition: D. Right ventricle and left auricle
A. Injury to left recurrent laryngeal nerve E. Right truck and right ventricle
B. Injury to right recurrent laryngeal nerve
C. Injury to trachea 171. The cartilaginous muscular and
D. Spasm of left main bronchus connective tissue component of primary
E. Spasm of right main bronchus bronchi develop from:
A. Somatopleuric mesoderm
165. The muscle pierced by the B. Endoderm
musculocutaneous nerve is: C. Ectoderm
A. Triceps brachii D. Splanchnopleuric mesoderm
B. Biceps brachii E. Neural crest cells
C. Brachialis
D. Coracobrachialis 172. A 35-year old man sustain a full thickness
E. Anconeus burn over the dorsum of forearm. A local flap
of normal skin and subcutaneous tissue is
166. The mediastinal pleura is supplied by the: grafted into the defect which remains
A. Intercostal nerves successful because of horizontally oriented
B. Sympathetic trunk vessels in which of the following layer:
C. Pulmonary plexus A. Basal layers
D. Phrenic nerve B. Deep dermis
C. Stratum corneum 179. In a section of lung tissue consisting
D. Stratum lucidum predominantly of alveoli and a tubule of
E. Subcutaneous fat about 2 mm in diameter that contains smooth
muscle and cartilage in its wall. The tubule is
173. During development of the body cavities. which one of the following:
Pleuro-pericardial folds contain which of the F. Alveolar duct
following structure/ structures: G. Alveolar sac
A. Common cardinal vein H. Bronchiole
B. Common cardinal vein and phrenic nerve I. Bronchus
C. Posterior cardinal vein and phrenic vein J. Trachea
D. Anterior and posterior cardinal vein
E. Phrenic nerve and cardia plexus 180. Which of the following separates the
parietal pleura from the thoracic wall:
174. Medullary cords in the medulla of lymph A. Suprapleural membrane
nodes are aggregation of: B. Anterior intercostal membrane
A. Lymphocytes C. Posterior intercostal membrane
B. T-lymphocytes and plasma cells D. Endothoracic fascia
C. B-lymphocytes and plasma cells E. Adipose tissue
D. Plasma cells
E. Reticular cells and lymphocytes 181. Which of the following artery supplies
blood to the breast:
175. Which of the following is not the feature A. Internal thoracic artery
of teratology of fallot: B. Lateral thoracic artery
A. Pulmonary stenosis C. Thoracoacromial artery
B. Overriding of aorta D. Superior thoracic artery
C. Ventricular septal defect E. Circumflex scapular artery
D. Atrial septal defect
E. Right ventricular hypertrophy 182. The entire lymphatic region of the spleen
is called as:
176. Which of the following accompanies the F. Malpighian corpuscle
anterior interventricular artery: G. Trabeculae
A. Coronary sinus H. White pulp
B. Small cardiac veins I. Red pulp
C. Great cardiac veins J. Cords of billroth
D. Anterior cardiac vein
E. Middle cardiac vein 183. The smooth portion of the left atrium is
derived from:
177. Foramen ovale is present in which of the A. The coronary sinus
following structure: B. Expansion of the primitive left atrium
A. Septum secondum C. Incorporation of the sinus venosus
B. Septum primum D. Cardiac loop
C. Septum sprium E. Incorporation of the pulmonary veins
D. Cristae terminalis
E. Left ventricle 184. The trachea bifurcates into right and left
bronchi at the level of:
178. Which of the following is the thickest A. Suprasternal notch
layer in the wall of veins: B. First rib
A. Internal elastic lamina C. Xiphoid process
B. Sub-endothelial connective tissues D. Sternal angle
C. Tunica adventitia E. Seventh cervical vertebrae
D. Tunica intima
E. Tunica media
185. What is the embryological origin of 191. The annular ligament of the proximal
ductus arteriosus: radioulnar joint forms a collar around the:
A. Pulmonary artery A. Radial notch of ulna
B. Left sixth aortic artery B. Head of the ulna
C. Right sixth aortic artery C. Head of the radius
D. Left atrium D. Ulnar notch of the radius
E. Dorsal aorta E. Neck of the radius
186. Which of the following structures passes 192. The wrist joint is a:
through the caval opening of the diaphragm: A. Synovial hinge joint
A. Azygous vein B. Synovial pivot joint
B. Inferior vena cava C. Synovial ball and socket joint
C. Thoracic duct D. Synovial ellipsoid joint
D. Aorta E. Synovial saddle joint
E. Esophagus
193. The characteristic feature of thymus is:
187. The intercostal nerve and blood vessels A. Principle cells
run: B. Chief cells
A. Between external and internal intercostal C. Lymph particles
muscles D. Hassal’s corpuscles
B. Between internal intercostal and inner most E. Lymph sinuses
intercostal muscles
C. Between external intercostal and inner most
intercostal muscles 194. Which of the following structure passes
D. Between innermost intercostal and through carpal tunnel:
endothoracic fascia A. Radial artery
E. Between parietal pleura and endothoracic B. Radial nerve
fascia C. Median nerve
D. Ulnar nerve
188. The spiral groove of humerus E. Ulnar artery
accommodates:
A. Median nerve 195. The thenar muscles are supplied by:
B. Ulnar nerve A. Ulnar nerve
C. Axillary nerve B. Median nerve
D. Radial nerve C. Radial nerve
E. Musculocutaneous nerve D. Anterior interroceous nerve
E. Posterior interrocous nerve
189. Which of the following is not included in
skin appendages: 196. The ulnar artery begins in the cubital fossa
A. Hair at the level of the:
B. Nail A. Trochlear notch of ulna
C. Sweat glands B. Coronoid process of ulna
D. Sebaceous glands C. Supinator cross of ulna
E. Arrector pilli muscle D. Neck of the radius
E. Head of the radius
190. Which of the following structure is not
present in superior mediastinum: 197. If the surgical neck of humerus is
A. Esophagus damaged, which of the following nerve is
B. Descending aorta most likely damaged:
C. Trachea A. Radial nerve
D. Sympathetic trunk B. Ulnar nerve
E. Thymus C. Axillary nerve
D. Median nerve
E. Musculocutaneous nerve
198. Inhaled foreign bodies most frequently 205. The entire lymphatic region of the spleen
enter in: is called as which one of the following:
A. Left lung A. Malpighian corpuscle
B. Left bronchus B. Trabeculae
C. Right bronchus C. White pulp
D. Esophagus D. Red pulp
E. Bronchioles E. Cords of the billorth
199. The manubrum sterni articulates with all 206. The injury to the upper trunk of the
of the following, except: brachial plexus is called:
A. Body of sternum A. Klumpke’s palsy
B. Clavicle B. Erb-Duchenne palsy
C. Xiphoid process C. Wrist drop
D. 1st costal cartilage D. Ape like hand
E. 2nd costal cartilage E. Claw hand
200. The wrist drop is caused by damage to: 207. Which one of the following is not the
A. Median nerve feature of teratology of fallot:
B. Radial nerve A. Atrial septal defect
C. Ulnar nerve B. Pulmonary stenosis
D. Axillary nerve C. Ventricular septal defect
E. Anterior interosseous nerve D. Overriding aorta
E. Right ventricular hypertrophy
201. The smooth portion of the left atrium is
derived from: 208. Which of the following bones are formed
A. The coronary sinus from somites:
B. Expansion of the primitive left atrium A. Skull bones totally
C. Incorporation of the sinus venosus B. Vertebrae only
D. Cardiac loop C. Ribs and vertebrae
E. Incorporation of the pulmonary veins D. Radius and ulna
E. Ribs
202. Air in the pleural cavity is known as:
A. Pleural effusion 209. The lymphoid organ covered by stratified
B. Hemopneumothorax squamous epithelium is:
C. Pyopneumothorax A. Spleen
D. Hydropneumothorax B. Liver
E. Pneumothorax C. Thymus
D. Palatine tonsil
203. Which of the following muscle is extensor E. Lymph node
of the elbow joint:
A. Brachialis 210. Which are the following are the false ribs:
B. Biceps brachi A. 11th and 12th
C. Brachio radialis B. Upper seven
D. Triceps brachi C. Lower five
E. Pronator teres D. 8th – 10th
E. 1st and 2nd
204. What is the embryological origin of ductus
arteriosus: 211. Which of the following contain simple
A. Pulmonary artery squamous epithelium in tunica intima walls
B. Left sixth aortic artery are less muscular having abundant elastic
C. Right sixth aortic artery tissue:
D. Left atrium A. Conducting artery
E. Dorsal aorta B. Distributing artery
C. Arteriole
D. Venules C. Posterior intercostal membrane
E. Muscular artery D. Endothoracic fascia
E. Adipose tissue
212. Most of the posterior intercostal arteries are
branches from: 219. The cartilaginous muscular and connective
A. Superior intercostal artery tissue component of primary bronchi develop
B. Inferior intercostal artery from:
C. Descending thoracic aorta A. Somatopleuric mesoderm
D. Internal thoracic artery B. Endoderm
E. Musculophrenic artery C. Ectoderm
D. Splanchnopleuric mesoderm
213. In the adult heart the fossa ovalis represent: E. Neural crest cells
A. Septum secondum
B. SeptumPrimum 220. The component from which ….. derived
C. Septum intermedium include all, except:
D. Septum spurium A. Septum transversum
E. Sinus venorum B. Pleuroperitoneal fluid
C. Dorsal mesentry
214. Which of the following arteries accompanies D. Ventral mesentry
the axillary nerve through quadrangular E. Cervical myotonica
space:
A. Subscapular artery 221. The nerve supply of parietal pleura comes
B. Anterior circumflex humeral artery from:
C. Posterior circumflex humeral artery A. Intercostal nerves
D. Profunda brachi artery B. Phrenic nerve
E. Supra scapular artery C. Vagus nerve
D. Recurrent laryngeal nerve
215. The muscle pierced by the E. Phrenic and intercostal nerves
musculocutaneous nerve is:
A. Triceps brachi 222. Slit like spaces between the costal and
B. Biceps brachi diaphragmatic parietal pleura are called:
C. Brachialis A. Costomediastinal recesses
D. Coracobrachialis B. Pleural recesses
E. Anconeus C. Pleural cavity
D. Costodiaphagmatic recesses
216. Medullary cords in the medullar of lymph E. Pulmonary ligament
nodes are aggregation of:
A. Lymphocytes 223. The air in pleural cavity when associated
B. T-lymphocytes and plasma cells with accumulation of serous fluid is known
C. B-lymphocytes and plasma cells as:
D. Plasma cells A. Pneumothorax
E. Reticular cells and lymphocytes B. Hydrothorax
C. Hemothorax
217. The right border of the heart is formed by: D. Hydropneumothorax
A. Right ventricle E. Pyopneumothorax
B. Right atrium
C. Right auricle 224. The airway which has a diameter of 0.5
D. Right atrium and right ventricle mm is called:
E. Right auricle and right ventricle A. Segmental bronchiole
B. Tertiary brochi
218. Which of the following separates the parietal C. Alveolar duct
pleura from the thoracic wall: D. Respiratory bronchiole
A. Suprapleural membrane E. Terminal bronchiole
B. Anterior intercostal membrane
225. In the lung the sympathetic efferent fibers 231. Clavicle is developed from:
produces: A. Hyaline cartilage
A. Vasodilatation B. Fibrocartilage
B. Broncho-dilatation C. One primary center of ossification
C. Broncho-constriction D. Two primary centers of ossification T
D. Broncho-dilatation and vasoconstriction E. Only two secondary centers
E. Produces more movement of lungs
232. The strongest ligament of sternoclavicular
226. The azygous vein enters the posterior joint is:
mediastinum: A. Interclavicular ligament
A. Behind the medial arcuate ligament B. Capsular ligament
B. Behind the lateral arcuate ligament C. Costoclavicular ligament T
C. Behind the median arcuate ligament D. Anterior sternoclavicular ligament
D. Through the caval opening E. Fibrocartilagenous disc
E. Through esophageal opening
233. The pectoralis major muscle arises from:
227. The sympathetic chain is receiving A. Upper 6 (six) ribs
preganglionic / white rami communicants B. Lower 6 (six) ribs
from: C. Middle 1/3rd of clavicle
A. Ventral horn of thoracic segments of spinal D. Only aponeurosis of external oblique muscle
cord E. Upper 6 (six) costal cartilages T
B. Lateral horn of thoracic segments of spinal
cord 234. The branch of axillary artery which
C. Lateral horn of lumbar segments of spinal pierces clavipectoral fascia is:
cord A. Superior thoracic artery
D. Lateral horn from T1 to L2 segments of B. Lateral thoracic artery
spinal cord C. Acromial artery
E. Dorsal horn of T1 to L2 segments of spinal D. Deltoid branch
cord E. Thoracoacromial artery T
228. Which of the following structure supplies 235. Which of the following nerve’s injury
pleura, pericardium and peritoneum: leads to winging of scapula:
A. Vagus nerve A. Thoracodorsal nerve
B. Sympathetic system B. Dorsal specular nerve
C. Parasympathetic system C. Long thoracic nerve T
D. Phrenic nerve D. Suprascapular nerve
E. Greater splanchnic nerve E. Axillary nerve
229. The thoracic duct drains lymph from all 236. Which group of axillary lymph nodes
of the following regions, except: drains lymph from breast and anterior
A. Both lower limbs abdominal wall:
B. Pelvic cavity A. Pectoral group T
C. Right side of thorax B. Subscapular group
D. Left side of head and neck C. Brachial group
E. Left upper limb D. Central group
E. Apical group
230. The esophagus passes through esophageal
opening along with: 237. Which of the following structure passes
A. Phrenic nerve through suprascapular foramen:
B. Vagus nerve A. Suprascapular artery
C. Azygous vein B. Suprascapular vein
D. Vagus nerve and esophageal vessels C. Suprascapular nerve
E. Cysterna chylli D. Subscapular artery
E. Dorsal scapular nerve
238. Which of the following muscles retract C. Azygous vein
scapula: D. Thoracic duct
A. Trapezius E. Sympathetic trunk
B. Trapezius and levator scapulae
C. Trapezius and latissmus dorsi 245. Mitral valve guards the:
D. Trapezius and two rhomboids A. Right atrioventricular orifice
E. Supraspinatus and infraspinatus B. Left atrioventricular orifice
C. Aortic orifice
239. Which of the following branches of D. Pulmonary orifice
brachial plexus supplies only one muscle: E. Caval orifice
A. Axillary nerve
B. Dorsal scapular nerve 246. All of the following drains into the
C. Thoracodorsal nerve T coronary sinus, except:
D. Suprascapular nerve A. Great cardiac veins
E. Musculocutaneous nerve B. Middle cardiac veins
C. Oblique vein of left atrium
240. Which of the following is the main muscle D. Anterior cardiac vein
to form anterior wall of axilla: E. Small cardiac vein
A. Pectoralis minor
B. Pectoralis major T 247. The azygous vein:
C. Serratus anterior A. Receives right superior intercostal veins
D. Subclavius B. Receives lower left intercostal veins
E. Teres minor C. Originates in thorax
D. Drains into the right atrium directly
241. The sternal angle is an important E. Lies on the left side of the aorta
landmark for:
A. Inferior border of heart 248. Regarding bronchopulmonary segments
B. Larynx bifurcates at this level all are correct, except:
C. Present at the 3rd costal cartilage A. It is a subdivision of a lung lobe
D. Arch of aorta begins and ends at this level B. It is surrounded by connective tissue
E. Junction of anterior and posterior mediastina C. It is pyramidal in shape and its apex towards
the lung surface
242. Regarding internal thoracic artery: D. It has a segmental bronchus
A. Arise from 3rd part of subclavian artery E. When diseased it can be removed surgically
B. Give rise to one anterior intercostal artery in as a structural unit
each intercostal space
C. Terminates in 5th intercostal space 249. Regarding ribs:
D. Finds by dividing into inferior epigastric and A. 3rd rib is atypical
mediastinal arteries B. Posterior end of 1st rib is larger, thicker and
E. Descends behind the costal cartilages pitter
C. The typical rib is grooved superiorly by the
243. A 5-year old girl has aspirated a coin. costal groove
Which of the following site of respiratory D. 12th rib moves with respiration
tract will be lodged: E. Articulates with corresponding vertebrae
A. Larynx only
B. Trachea
C. Right main bronchus 250. Cords of billroth are found in:
D. Left main bronchus A. Tonsil
E. Terminal bronchiole B. Appendix
C. Spleen
244. Which of the following is not a posterior D. Lymph node
mediastinal content: E. Thymus
A. Trachea
B. Esophagus
251. Which of the following is not content of 257. A patient is unable to flex the proximal
pericardium: interphalangeal joints as a result of paralysis
A. Ascending aorta of which of the following muscles:
B. Superior vena cava A. Palmar interossei
C. Inferior vena cava B. Flexor digitorum profundus
D. Descending aorta C. Dorsal interossei
E. Pulmonary veins D. Flexor digitorum superficialis T
E. Lumbricals
252. The fetal part of placenta is formed by:
A. Decidua basalis 258. A construction worker suffers a
B. Decidua capsularis destructive injury of the structure related to
C. Chorion leave the anatomical snuff box. Which of the
D. Chorion frondosum following structures most likely be damaged:
E. Abembryonic pole of embryo A. Triquetral bone
B. Trapezoid bone
253. Renal agenesis may lead to: C. Extensor indicis tendon
A. Polyhydramnios D. Abductor pollicis brevis tendon
B. Oligohydramnios E. Radial artery T
C. Fetal uremia
D. Fetal acidosis 259. A 27-year old patient is given radiopaque
E. Premature labour dye in preparation for an arteriogram of the
radial artery that:
254. Following are the features regarding A. Enters the hand through the carpal tunnel
lymph node: B. Accompanies the posterior interosseous
A. Germinal centers are found in superficial nerve in the forearm T
cortex composed of B-cells C. Is the principle source of blood to the
B. Paracortex is composed of T-cells superficial palmar arterial arch
C. Plasma cells are the major cells in medullary D. Has the princeps pollicis artery as one of its
cords branches
D. Lymphoid nodules are present in the cortex E. Runs between the flexor digitorum
E. Majority of lymphocytes gain entry via the superficialis and profundus muscles
afferent lymphatic vessels
260. A patient comes to the doctor with arm
255. Regarding layers of skin: unflexed and wrist flexed it is due to:
A. Basal cells are cuboidal or low columnar A. Radial nerve injury T
B. Basement membrane and basal cells are B. Ulnar nerve injury
attached by hemi-desmosomes C. Median nerve injury
C. Langerhans cells are present in all layers of D. Axillary nerve injury
epidermis E. None of above
D. Fibers are more in papillary as compare to
reticula dermis 261. Injury to the anterior interosseous nerve
E. Keratinocytes are attached by desmosomes could result in paralysis of which of the
following muscles:
256. A patient has loss of sensation on the A. Flexor pollicis longus and brevis
lateral aspect of the forearm. Through which B. Flexor pollicis longus and opponens pollicis
of the following muscles does the nerve that C. Flexor digitorum profundus and pronator
supplies this region pass: quadratus T
A. Coracobrachialis T D. Flexor digitorum profundus and superficialis
B. Flexor carpi ulnaris E. Flexor pollicis brevis and pronator quadratus
C. Flexor digitorum superficialis
D. Pronator teres
E. Supinator
262. A lesion of the median nerve produces B. Its short head takes origin in common with
complete paralysis of all of the following coracobrachialis
muscles, except: C. It is a powerful supinator
A. Flexor pollicis longus D. Its tendon has brachial artery on its lateral
B. Flexor digitorum superficialis side in the cubital fossa T
C. Pronator quadratus E. It is innervated by the musculocutaneous
D. Flexor digitorum profundus T nerve
E. Pronator teres
268. Regarding triceps muscle:
263. All of the following pairs are correctly A. Its long head takes origin from the
matched, except: supraglenoid tubercle
A. Ulnar artery --- Ulnar nerve B. Its lateral head takes origin from back of the
B. Radial artery --- Deep branch of radial nerve shaft of humerus below the radial groove
T C. Its medial head takes origin from back of the
C. Brachial artery --- Median nerve shaft of humerus above the radial groove
D. Anterior interosseous artery --- Anterior D. Its long head receives two branches from the
interosseous nerve radial nerve
E. Posterior interosseous artery --- Deep branch E. The nerve to medial head of triceps also
of radial nerve supplies the anconeus muscle
264. All are the branches of the ulnar artery, 269. Regarding the radial nerve:
except: A. It is the continuation of the medial cord of
A. Anterior ulnar recurrent artery brachial plexus
B. Posterior ulnar recurrent artery B. It passes through the quadrangular space
C. Common interosseous artery T C. It accompanies profunda brachi artery in the
D. Palmar carpal artery radial groove T
E. Arteria princes pollicis D. It pierces the medial intermuscular septum
E. It partly innervates the coracobrachialis
265. All of the following pairs are correctly muscle
matched, except:
A. Supinator muscle --- Deep branch of radial 270. Regarding the anastomosis around the
nerve elbow joint, all are true, except:
B. Extensor carpi radialis longus muscle --- A. Superior ulnar collateral artery --- Anterior
Main trunk of radial nerve ulnar recurrent artery
C. Extensor carpi radialis brevis muscle --- B. Inferior ulnar collateral artery --- Posterior
Main trunk of radial nerve T ulnar recurrent artery
D. Brachioradialis muscle --- Main trunk of C. Middle collateral artery --- Interosseous
radial nerve recurrent artery
E. Extensor digitorum --- Deep branch of radial D. Radial collateral artery --- Radial recurrent
nerve artery
E. Superior ulnar collateral artery --- Profunda
266. Which of the following is true regarding brachi artery
the structures piercing the medial
intermuscular septum: 271. Regarding the cubital fossa all are true,
A. Ulnar nerve except:
B. Superior ulnar collateral artery A. Its medial boundary is formed by the
C. Ulnar collateral nerve pronator teres muscle
D. A+B+C T B. Its lateral boundary is formed by the
E. None of the above brachioradialis muscle
C. It has the median nerve lying on the lateral
267. Regarding biceps brachi all are true, side of the brachial artery T
except: D. Medial part of its floor is formed by
A. Its long head is intracapsular brachialis
E. Lateral part of its floor is formed by 277. On the basis of the examination at her
supinator doctor’s office, a patient is told that her
parasympathetic nerves are damaged. Which
272. All of the following matchings are true, of the following muscles would most likely be
except: affected?
A. Ulnar nerve --- Medial epicondyle A. Muscles in the hair follicles
B. Radial nerve --- Spiral groove B. Muscles in blood vessels
C. Axillary nerve --- Surgical neck C. Muscles that act at the elbow joint
D. Musculocutaneous nerve --- D. Muscles in the gastrointestinal (GI) tract T
Coracobrachialis E. Muscles enclosed by epimysium
E. Median nerve --- Lateral epicondyle T
278. A 46-year-old male patient with high
273. Regarding the shoulder joint all are true, blood pressure was examined in the
except: emergency department, and his physician
A. Transverse humeral ligament bridges the found a leakage of blood from the blood
lesser and greater tubercles vessel that normally carries richly oxygenated
B. Subscapular bursa is continuous with the blood. Which of the following vessels would
synovial membrane of the joint most likely be damaged?
C. Subacromial bursa is continuous with the A. Superior vena cava
synovial membrane of the joint B. Pulmonary arteries
D. Its inferior dislocation is common C. Pulmonary veins T
E. It is a typical synovial joint T D. Coronary sinus
274. All are the branches of radial nerve, 279. A 16-year-old patient received a stab
except: wound, and axons of the general somatic
A. Posterior cutaneous nerve of arm efferent (GSE) neurons to the shoulder
B. Posterior cutaneous nerve of forearm muscles were severed. The damaged axons:
C. Upper lateral cutaneous nerve of arm T A. Would carry impulses toward the cell bodies
D. Nerve to medial head of triceps B. Would carry impulses away from the cell
E. Nerve to lateral head of triceps Bodies T
C. Would carry pain impulses
275. All of the following pairs are correct, D. Are several in numbers for multipolar
except: neurons
A. Axillary nerve --- Posterior circumflex E. Are found primarily in the gray matter
humeral artery
B. Radial nerve --- Profunda brachii artery 280. A 16-year-old patient received a
C. Ulnar nerve --- Superior ulnar collateral laceration of the posterior intercostal nerves
artery by a penetrated knife blade. A pathologist
D. Median nerve --- Brachial artery obtained needle biopsy tissues and observed
E. Radial nerve --- Middle collateral artery T numerous degenerated cell bodies of the
unipolar or pseudounipolar neurons. Which
276. A 22-year-old man presented to his family of the following structures would most likely
physician with a laceration of the fibrous provide the abnormal cell morphology?
sheets or bands that cover his body under the A. Ventral horn of the spinal cord
skin and invest the muscles. Which of the B. Lateral horn of the spinal cord
following structures would most likely be C. Dorsal horn of the spinal cord
injured? D. Dorsal root ganglion T
A. Tendon E. Sympathetic chain ganglion
B. Fascia T
C. Synovial tendon sheath
D. Aponeurosis
E. Ligament
281. A 19-year-old college student came to his C. Large intestine T
doctor’s office for a neurologic examination. D. Liver
His physician told him that normally E. Pancreas
synapses are absent in or on which of the
following structures? 286. A 16-year-old girl with urinary diseases
A. Anterior horn of the spinal cord comes to a local hospital. Her urologist’s
B. Dorsal root ganglia T examination and laboratory test results reveal
C. Sympathetic chain ganglia that she has difficulty in removing wastes
D. Dendrites from the blood and in producing urine.
E. Cell bodies Which of the following organs may have
abnormal functions?
282. A 27-year-old woman involved in a car A. Ureter
accident is brought into the emergency B. Spleen
department. Her magnetic resonance imaging C. Urethra
reveals that she has a laceration of the spinal D. Bladder
cord at the L4 spinal cord level. Which of the E. Kidney T
following structures would you expect to be
intact? 287. A 53-year-old man with a known history
A. Dorsal horn of emphysema is examined in the emergency
B. Lateral horn T department. Laboratory findings along with
C. Ventral horn examination indicate that the patient is
D. Gray matter unable to exchange oxygen in the air and
E. White matter carbon dioxide in the blood. This exchange
occurs in which portion of the respiratory
283. A 33-year-old male patient complains of system?
feeling severe pain when he tries to turn his A. Bronchi
neck. A physician realizes that the problem is B. Alveolar (air) sac T
in his pivot (trochoid) joint. Which of the C. Nasal cavity
following joints would most likely be D. Larynx
examined? E. Trachea
A. Atlantooccipital joint
B. Atlantoaxial joint T 288. A 26-year-old woman has an
C. Carpometacarpal joint amenorrhea, followed by uterine bleeding,
D. Proximal tibiofi bular joint pelvic pain, and pelvic mass. Her obstetrician
E. Intervertebral disks performed a thorough examination, and the
patient was diagnosed as having an ectopic
284. A patient presents with a loss of pregnancy. Which of the following organs is
sensation to the skin over the shoulder. Injury most likely to provide a normal site of
to which of the following nerve cells would fertilization?
most likely affect the conduction of sensory A. Fundus of the uterus
information to the central nervous system? B. Ampulla of the uterine tube T
A. Multipolar neurons C. Fimbriae
B. Bipolar neurons D. Infundibulum of the uterine tube
C. Unipolar or pseudounipolar neurons T E. Body of the uterus
D. Neurons in the ventral horn
E. Neurons in sympathetic chain ganglia 289. A 29-year-old woman with abdominal
pain was admitted to a local hospital, and
285. A 7-year-old girl comes to the emergency examination shows that a retroperitoneal
department with severe diarrhea. Tests show infection is affecting a purely endocrine
that the diarrhea is due to decreased capacity gland. Which of the following structures is
of normal absorption in one of her organs. infected?
Which of the following organs is involved? A. Ovary
A. Stomach B. Suprarenal gland T
B. Gallbladder C. Pancreas
D. Liver B. Loss of skin sensation on the lateral side of
E. Stomach the foot
C. Loss of skin sensation over the greater
290. A 36-year-old woman received a first trochanter
degree burn on her neck, arm, and forearm D. Paralysis of the vastus lateralis muscle T
from a house fire. Which of the following skin E. Paralysis of the tensor fasciae latae
structures or functions is most likely
damaged or impaired? 295. A 47-year-old woman is unable to invert
A. GSE nerves her foot after she stumbled on her driveway.
B. Parasympathetic general visceral efferent Which of the following nerves are most likely
nerves injured?
C. Trophic hormone production A. Superficial and deep peroneal
D. Exocrine gland secretion T B. Deep peroneal and tibial T
E. Vitamin A production C. Superficial peroneal and tibial
D. Medial and lateral plantar
291. A 27-year-old patient exhibits a loss of E. Obturator and tibial
skin sensation and paralysis of muscles on the
plantar aspect of the medial side of the foot. 296. A 22-year-old patient is unable to
Which of the following nerves is most likely “unlock” the knee joint to permit flexion of
damaged? the leg. Which of the following muscles is
A. Common peroneal most likely damaged?
B. Tibial T A. Rectus femoris
C. Superficial peroneal B. Semimembranosus
D. Deep peroneal C. Popliteus T
E. Sural D. Gastrocnemius
E. Biceps femoris
292. A patient with a deep knife wound in the
buttock walks with a waddling gait that is 297. A patient presents with sensory loss on
characterized by the pelvis falling toward one adjacent sides of the great and second toes
side at each step. Which of the following and impaired dorsiflexion of the foot. These
nerves is damaged? signs probably indicate damage to which of
A. Obturator nerve the following nerves?
B. Nerve to obturator internus A. Superficial peroneal
C. Superior gluteal nerve T B. Lateral plantar
D. Inferior gluteal nerve C. Deep peroneal T
E. Femoral nerve D. Sural
E. Tibial
293. A patient is unable to prevent anterior
displacement of the femur on the tibia when 298. A motorcyclist falls from his bike in an
the knee is flexed. Which of the following accident and gets a deep gash that severs the
ligaments is most likely damaged? superficial peroneal nerve near its origin.
A. Anterior cruciate Which of the following muscles is paralyzed?
B. Fibular collateral A. Peroneus longus T
C. Patellar B. Extensor hallucis longus
D. Posterior cruciate T C. Extensor digitorum longus
E. Tibial collateral D. Peroneus tertius
E. Extensor digitorum brevis
294. A 41-year-old man was involved in a fight
and felt weakness in extending the knee joint. 299. A 67-year-old patient has been given a
On examination, he was diagnosed with a course of antibiotics by gluteal intramuscular
lesion of the femoral nerve. Which of the injections after a major abdominal surgery.
following symptoms would be a result of this To avoid damaging the sciatic nerve during
nerve damage? an injection, the needle should be inserted
A. Paralysis of the psoas major muscle into which of the following areas?
A. Over the sacrospinous ligament muscles was paralyzed as a result of this
B. Midway between the ischial tuberosity and accident?
the lesser trochanter A. Semitendinosus
C. Midpoint of the gemelli muscles B. Sartorius
D. Upper lateral quadrant of the gluteal region C. Gracilis
T D. Quadriceps femoris T
E. Lower medial quadrant of the gluteal Region E. Biceps femoris
300. A 20-year-old patient cannot flex and 305. A patient experiences weakness in
medially rotate the thigh while running and dorsiflexing and inverting the foot. Which of
climbing. Which of the following muscles is the following muscles is damaged?
most likely damaged? A. Peroneus longus
A. Semimembranosus B. Peroneus brevis
B. Sartorius C. Tibialis anterior T
C. Rectus femoris D. Extensor digitorum longus
D. Vastus intermedius E. Peroneus tertius
E. Tensor fasciae latae T
306. A 62-year-old woman slips and falls on
301. A 21-year-old man was involved in a the bathroom floor. As a result, she has a
motorcycle accident, resulting in destruction posterior dislocation of the hip joint and a
of the groove in the lower surface of the fracture of the neck of the femur Rupture of
cuboid bone. Which of the following muscle the ligamentum teres capitis femoris may lead
tendons is most likely damaged? to damage to a branch of which of the
A. Flexor hallucis longus following arteries?
B. Peroneus brevis A. Medial circumflex femoral
C. Peroneus longus T B. Lateral circumflex femoral
D. Tibialis anterior C. Obturator T
E. Tibialis posterior D. Superior gluteal
E. Inferior gluteal
302. A construction worker falls feet first from
a roof. He sustains a fracture of the groove on 307. A 62-year-old woman slips and falls on
the undersurface of the sustentaculum tali of the bathroom floor. As a result, she has a
the calcaneus bone. Which of the following posterior dislocation of the hip joint and a
muscle tendons is most likely torn? fracture of the neck of the femur Fracture of
A. Flexor digitorum brevis the neck of the femur results in avascular
B. Flexor digitorum longus necrosis of the femoral head, probably
C. Flexor hallucis brevis resulting from lack of blood supply from
D. Flexor hallucis longus T which of the following arteries?
E. Tibialis posterior A. Obturator
B. Superior gluteal
303. A thoracic surgeon is going to collect a C. Inferior gluteal
portion of the greater saphenous vein for D. Medial femoral circumflex T
coronary bypass surgery. He has observed E. Lateral femoral circumflex
that this vein runs:
A. Posterior to the medial malleolus 308. A 62-year-old woman slips and falls on
B. Into the popliteal vein the bathroom floor. As a result, she has a
C. Anterior to the medial condyles of the tibia posterior dislocation of the hip joint and a
and femur fracture of the neck of the femur. If the
D. Superficial to the fascia lata of the thigh T acetabulum is fractured at its posterosuperior
E. Along with the femoral artery margin by dislocation of the hip joint, which
of the following bones could be involved?
304. A 52-year-old woman slipped and fell and A. Pubis
now complains of being unable to extend her B. Ischium
leg at the knee joint. Which of the following C. Ilium T
D. Sacrum C. Sartorius
E. Head of the femur D. Biceps femoris (long head)
E. Rectus femoris
309. A 62-year-old woman slips and falls on
the bathroom floor. As a result, she has a 313. A patient is involved in a motorcycle
posterior dislocation of the hip joint and a wreck that results in avulsion of the skin over
fracture of the neck of the femur. The woman the anterolateral leg and ankle. Which of the
experiences weakness when abducting and following structures is most likely destroyed
medially rotating the thigh after this accident. with this type of injury?
Which of the following muscles is most likely A. Deep peroneal nerve
damaged? B. Extensor digitorum longus muscle tendon
A. Piriformis C. Dorsalis pedis artery
B. Obturator internus D. Great saphenous vein
C. Quadratus femoris E. Superficial peroneal nerve T
D. Gluteus maximus
E. Gluteus minimus T 314. A knife wound penetrates the superficial
vein that terminates in the popliteal vein.
310. A 62-year-old woman slips and falls on Bleeding occurs from which of the following
the bathroom floor. As a result, she has a vessels?
posterior dislocation of the hip joint and a A. Posterior tibial vein
fracture of the neck of the femur. The woman B. Anterior tibial vein
undergoes hip surgery. If all of the arteries C. Peroneal vein
that are part of the cruciate anastomosis of D. Great saphenous vein
the upper thigh are ligated, which of the E. Lesser saphenous vein T
following arteries maintains blood flow?
A. Medial femoral circumflex 315. A 10-year-old boy falls from a tree house.
B. Lateral femoral circumflex The resultant heavy compression of the sole
C. Superior gluteal T of his foot against the ground caused a
D. Inferior gluteal fracture of the head of the talus. Which of the
E. First perforating following structures is unable to function
normally?
311. A 34-year-old woman sustains a deep cut A. Transverse arch
on the dorsum of the foot just distal to her B. Medial longitudinal arch T
ankle joint by a falling kitchen knife. A C. Lateral longitudinal arch
physician in the emergency department has D. Tendon of the peroneus longus
ligated the dorsalis pedis artery proximal to E. Long plantar ligament
the injured area. Which of the following
conditions most likely occurs as a result of the 316. A 24-year-old woman complains of
injury? weakness when she extends her thigh and
A. Ischemia in the peroneus longus muscle rotates it laterally. Which of the following
B. Aneurysm in the plantar arterial arch muscles is paralyzed?
C. Reduction of blood flow in the medial tarsal A. Obturator externus
artery T B. Sartorius
D. Low blood pressure in the anterior tibial C. Tensor fasciae latae
artery D. Gluteus maximus T
E. High blood pressure in the arcuate artery E. Semitendinosus
312. A patient experiences paralysis of the 317. A patient with hereditary blood clotting
muscle that originates from the femur and problems presents with pain in the back of
contributes directly to the stability of the knee her knee. An arteriogram reveals a blood clot
joint. Which of the following muscles is in the popliteal artery at its proximal end.
involved? Which of the following arteries will allow
A. Vastus lateralis T blood to reach the foot?
B. Semimembranosus A. Anterior tibial
B. Posterior tibial Radiographic examination reveals a fracture
C. Peroneal of the head and neck of the fibula. After
D. Lateral circumflex femoral T injury to this nerve, which of the following
E. Superior medial genicular muscles could be paralyzed?
A. Gastrocnemius
318. A 72-year-old woman complains of a B. Popliteus
cramp-like pain in her thigh and leg. She was C. Extensor hallucis longus T
diagnosed as having a severe intermittent D. Flexor digitorum longus
claudication. Following surgery, an infection E. Tibialis posterior
was found in the adductor canal, damaging
the enclosed structures. Which of the 323. A 20-year-old college student receives a
following structures remains intact? severe blow on the inferolateral side of the
A. Femoral artery left knee joint while playing football.
B. Femoral vein Radiographic examination reveals a fracture
C. Saphenous nerve of the head and neck of the fibula. If the
D. Great saphenous vein T lateral (fibular) collateral ligament is torn by
E. Nerve to the vastus medialis this fracture, which of the following
conditions may occur?
319. A basketball player was hit in the thigh A. Abnormal passive abduction of the extended
by an opponent’s knee. Which of the leg
following arteries is likely to compress and B. Abnormal passive adduction of the extended
cause ischemia because of the bruise and leg T
damage to the extensor muscles of the leg? C. Anterior displacement of the femur on the
A. Popliteal tibia
B. Deep femoral D. Posterior displacement of the femur on the
C. Anterior tibial T tibia
D. Posterior tibial E. Maximal flexion of the leg
E. Peroneal
324. A 20-year-old college student receives a
320. An elderly woman fell at home and severe blow on the inferolateral side of the
fractured the greater trochanter of her left knee joint while playing football.
femur. Which of the following muscles would Radiographic examination reveals a fracture
continue to function normally? of the head and neck of the fibula. Which of
A. Piriformis the following arteries could also be damaged
B. Obturator internus by this fracture?
C. Gluteus medius A. Popliteal
D. Gluteus maximus T B. Posterior tibial
E. Gluteus minimus C. Anterior tibial T
D. Peroneal
321. A 20-year-old college student receives a E. Lateral inferior genicular
severe blow on the inferolateral side of the
left knee joint while playing football. 325. A 20-year-old college student receives a
Radiographic examination reveals a fracture severe blow on the inferolateral side of the
of the head and neck of the fibula. Which of left knee joint while playing football.
the following nerves is damaged? Radiographic examination reveals a fracture
A. Sciatic of the head and neck of the fibula. Which of
B. Tibial the following conditions would occur from
C. Common peroneal T this fracture?
D. Deep peroneal A. Ischemia in the gastrocnemius
E. Superficial peroneal B. Loss of plantar flexion
C. Trendelenburg’s sign
322. A 20-year-old college student receives a D. Anterior tibial compartment syndrome T
severe blow on the inferolateral side of the E. Flat foot
left knee joint while playing football.
326. A construction worker is hit on the leg 331. While playing football, a 19-year-old
with a concrete block and is subsequently college student receives a twisting injury to
unable to plantar flex and invert his foot. his knee when being tackled from the lateral
Which of the following muscles is most likely side. Which of the following conditions most
damaged? likely has occurred?
A. Extensor digitorum longus A. Tear of the medial meniscus T
B. Tibialis anterior B. Ruptured fibular collateral ligament
C. Tibialis posterior T C. Tenderness on pressure along the fibular
D. Peroneus longus collateral ligament
E. Peroneus brevis D. Injury of the posterior cruciate ligament
E. Swelling on the back of the knee joint
327. The obturator nerve and the sciatic (tibial
portion) nerve of a 15-year-old boy are 332. A patient has weakness when flexing both
transected as a result of a motorcycle her thigh and leg. Which of the following
accident. This injury would result in complete muscles is most likely injured?
paralysis of which of the following muscles? A. Rectus femoris
A. Rectus femoris B. Semitendinosus
B. Biceps femoris, short head C. Biceps femoris
C. Pectineus D. Sartorius T
D. Adductor magnus T E. Adductor longus
E. Sartorius
333. A 35-year-old man has difficulty in
328. A 24-year-old woman presents to her dorsiflexing the foot. Which of the following
physician with weakness in flexing the hip muscles is most likely damaged?
joint and extending the knee joint. Which A. Tibialis posterior
muscle is most likely involved in this B. Flexor digitorum longus
scenario? C. Tibialis anterior T
A. Sartorius D. Peroneus longus
B. Gracilis E. Peroneus brevis
C. Rectus femoris T
D. Vastus medialis 334. An injury to the leg of a golfer results in
E. Semimembranosus loss of the ability to invert the foot. Which of
the following muscles is most likely
329. A 17-year-old boy was stabbed during a paralyzed?
gang fight resulting in transection of the A. Tibialis posterior T
obturator nerve. Which of the following B. Peroneus longus
muscles is completely paralyzed? C. Peroneus brevis
A. Pectineus D. Peroneus tertius
B. Adductor magnus E. Extensor digitorum longus
C. Adductor longus T
D. Biceps femoris 335. An orthopedic surgeon ligates the
E. Semimembranosus posterior tibial artery at its origin. Which of
the following arteries has no blood flow
330. A 32-year-old carpenter fell from the immediately after the ligation?
roof. The lateral longitudinal arch of his foot A. Peroneal T
was flattened from fracture and displacement B. Dorsalis pedis
of the keystone for the arch. Which of the C. Superior medial genicular
following bones is damaged? D. Anterior tibial
A. Calcaneus E. Descending genicular
B. Cuboid bone T
C. Head of the talus
D. Medial cuneiform
E. Navicular bone
336. Before knee surgery, a surgeon ligates A. Tibialis posterior
arteries participating in the anastomosis B. Gastrocnemius T
around the knee joint. Which of the following C. Soleus
arteries is most likely spared? D. Peroneus longus
A. Lateral superior genicular E. Flexor digitorum longus
B. Medial inferior genicular
C. Descending branch of the lateral femoral 341. A 28-year-old basketball player falls
circumflex while rebounding and is unable to run and
D. Saphenous branch of the descending jump. On physical examination, he has pain
genicular T and weakness when extending his thigh and
E. Anterior tibial recurrent flexing his leg. Which muscle involved in both
movements is most likely injured?
337. A 25-year-old gladiator sustains a A. Short head of biceps femoris
penetrating injury that severs the superficial B. Adductor magnus
peroneal nerve. This will most likely cause C. Semitendinosus T
paralysis of which of the following muscles? D. Sartorius
A. Peroneus tertius E. Gracilis
B. Peroneus brevis T
C. Flexor hallucis longus 342. A 52-year-old woman comes to an
D. Tibialis anterior orthopedic surgeon complaining of an
E. Tibialis posterior uncomfortable feeling in her knee and ankle
joints. After a thorough examination, she is
338. A patient presents with a thrombosis in diagnosed as having arthritis with mild
the popliteal vein. This thrombosis most discomfort with passive movements. The
likely causes reduction of blood flow in which muscles acting at the ankle joint appear
of the following veins? normal with good strength. Which muscle
A. Greater saphenous can both dorsiflex and invert her foot?
B. Lesser saphenous A. Peroneus longus
C. Femoral T B. Peroneus brevis
D. Posterior tibial C. Peroneus tertius
E. Anterior tibial D. Extensor hallucis longus T
E. Extensor digitorum longus
339. A 21-year-old tennis player comes to an
emergency room and complains of pain in the 343. Which of the following is a major
knee joint. On examination, he has an characteristic of meiosis I?
infection inside the knee joint capsule but A. Splitting of the centromere
outside the synovial cavity. Which of the B. Pairing of homologous chromosomes T
following structures is preserved from this C. Reducing the amount of DNA to 1N
infection? D. Achieving the diploid number of
A. Anterior cruciate ligament chromosomes
B. Posterior cruciate ligament E. Producing primordial germ cells
C. Lateral meniscus
D. Lateral collateral ligament T 344. A normal somatic cell contains a total of
E. Medial meniscus 46 chromosomes. What is the normal
complement of chromosomes found in a
340. A 14-year-old gymnastic silver medalist sperm?
falls from the parallel bar and complains of A. 22 autosomes plus a sex chromosome T
pains from the knee and ankle joints. On B. 23 autosomes plus a sex chromosome
physical examination, her physician found C. 22 autosomes
that the muscle responsible for flexing the leg D. 23 autosomes
at the knee joint and plantar flexing the foot E. 23 paired autosomes
is severely weakened. Which of the following
muscles involved in both movements was
most likely damaged in this accident?
345. Which of the following describes the 351. In the production of male gametes, which
number of chromosomes and amount of DNA of the following cells remains dormant for 12
in a gamete? years?
A. 46 chromosomes, 1N A. Primordial germ cell T
B. 46 chromosomes, 2N B. Primary spermatocyte
C. 23 chromosomes, 1N T C. Secondary spermatocyte
D. 23 chromosomes, 2N D. Spermatid
E. 23 chromosomes, 4N E. Sperm
346. Which of the following chromosome 352. Approximately how many sperm will be
compositions in a sperm normally results in ejaculated by a normal fertile male during
the production of a genetic female if sexual intercourse?
fertilization occurs? A. 10 million
A. 23 homologous pairs of chromosomes B. 20 million
B. 22 homologous pairs of chromosomes C. 35 million
C. 23 autosomes plus an X chromosome D. 100 million
D. 22 autosomes plus a Y chromosome E. 350 million T
E. 22 autosomes plus an X chromosome T
353. A young woman enters puberty with
347. In the process of meiosis, DNA replication approximately 40,000 primary oocytes in her
of each chromosome occurs, forming a ovary. About how many of these primary
structure consisting of two sister chromatids oocytes will be ovulated over the entire
attached to a single centromere. What is this reproductive life of the woman?
structure? A. 40,000
A. A duplicated chromosome T B. 35,000
B. Two chromosomes C. 480 T
C. A synapsed chromosome D. 48
D. A crossover chromosome E. 12
E. A homologous pair
354. Fetal sex can be diagnosed by noting the
348. All primary oocytes are formed by presence or absence of the Barr body in cells
A. week 4 of embryonic life obtained from the amniotic fluid. What is the
B. month 5 of fetal life T etiology of the Barr body?
C. birth A. Inactivation of both X chromosomes
D. month 5 of infancy B. Inactivation of homologous chromosomes
E. puberty C. Inactivation of one Y chromosome
D. Inactivation of one X chromosome T
349. When does formation of primary E. Inactivation of one chromatid
spermatocytes begin?
A. During week 4 of embryonic life 355. How much DNA does a primary
B. During month 5 of fetal life spermatocyte contain?
C. At birth A. 1N
D. During month 5 of infancy B. 2N
E. At puberty T C. 4N T
D. 6N
350. In the production of female gametes, E. 8N
which of the following cells can remain
dormant for 12 to 40 years? 356. During meiosis, pairing of homologous
A. Primordial germ cell chromosomes occurs, which permits large
B. Primary oocyte T segments of DNA to be exchanged. What is
C. Secondary oocyte this process called?
D. First polar body A. Synapsis
E. Second polar body B. Nondisjunction
C. Alignment
D. Crossing over T 361. When does a secondary oocyte complete
E. Disjunction its second meiotic division to become a
mature ovum?
357. During ovulation, the secondary oocyte A. At ovulation
resides at what specific stage of meiosis? B. Before ovulation T
A. Prophase of meiosis I C. At fertilization
B. Prophase of meiosis II D. At puberty
C. Metaphase of meiosis I E. Before birth
D. Metaphase of meiosis II T
E. Meiosis is completed at the time of 362. How soon after fertilization occurs within
Ovulation the uterine tube does the blastocyst begin
implantation?
358. Concerning maturation of the female A. Within minutes
gamete (oogenesis), when do the oogonia B. By 12 hours
enter meiosis I and undergo DNA replication C. By day 1
to form primary oocytes? D. By day 2
A. During fetal life T E. By day 7 T
B. At birth
C. At puberty 363. Where does the blastocyst normally
D. With each ovarian cycle implant?
E. Following fertilization A. Functional layer of the cervix
B. Functional layer of the endometrium T
359. Where do primordial germ cells initially C. Basal layer of the endometrium
develop? D. Myometrium
A. In the gonads at week 4 of embryonic E. Perimetrium
development
B. In the yolk sac at week 4 of embryonic 364. Which of the following events is involved
development T in the cleavage of the zygote during week 1 of
C. In the gonads at month 5 of embryonic development?
development A. A series of meiotic divisions forming
D. In the yolk sac at month 5 of embryonic blastomeres
development B. Production of highly differentiated
E. In the gonads at puberty blastomeres
C. An increased cytoplasmic content of
360. A 20-year-old woman presents at the blastomeres
emergency department with severe D. An increase in size of blastomeres
abdominal pain on the right side with signs of E. A decrease in size of blastomeres T
internal bleeding. She indicated that she has
been sexually active without contraception 365. Which of the following structures must
and missed her last menstrual period. Based degenerate for blastocyst implantation to
on this information, which of the following occur?
disorders must be included as an option in the A. Endometrium in progestational phase
diagnosis? B. Zona pellucida T
A. Ovarian cancer C. Syncytiotrophoblast
B. Appendicitis D. Cytotrophoblast
C. Normal pregnancy E. Functional layer of the endometrium
D. Ectopic tubal pregnancy T
E. Toxemia of pregnancy 366. Which of the following is the origin of the
mitochondrial DNA of all human adult cells?
A. Paternal only
B. Maternal only T
C. A combination of paternal and maternal
D. Either paternal or maternal
E. Unknown origin
D. Exocoelomic membrane and
367. Individual blastomeres were isolated from syncytiotrophoblast
a blastula at the 4-cell stage. Each blastomere E. Exocoelomic membrane and cytotrophoblast
was cultured in vitro to the blastocyst stage T
and individually implanted into four
pseudopregnant foster mothers. Which of the 372. During week 2 of development, the
following would you expect to observe 9 embryoblast receives its nutrients via
months later? A. Diffusion T
A. Birth of one baby B. Osmosis
B. Birth of four genetically different babies C. Reverse osmosis
C. Birth of four genetically identical babies T D. Fetal capillaries
D. Birth of four grotesquely deformed babies E. Yolk sac nourishment
E. No births
373. The prochordal plate marks the site of the
368. Embryonic carcinoma (EC) cells were future
isolated from a yellow-coated mouse with a A. Umbilical cord
teratocarcinoma. The EC cells were then B. Heart
microinjected into the inner cell mass of a C. Mouth T
blastocyst isolated from a black-coated D. Anus
mouse. The blastocyst was subsequently E. Nose
implanted into the uterus of a whitecoated
foster mouse. Which of the following would 374. Which of the following are components of
be observed after full-term pregnancy? the definitive chorion?
A. A yellow-coated offspring A. Extraembryonic somatic mesoderm and
B. A black-coated offspring epiblast
C. A white-coated offspring B. Extraembryonic somatic mesoderm and
D. A yellow- and black-coated offspring T cytotrophoblast
E. A yellow- and white-coated offspring C. Extraembryonic somatic mesoderm and
syncytiotrophoblast
369. In oogenesis, which of the following D. Extraembryonic somatic mesoderm,
events occurs immediately following the cytotrophoblast, and syncytiotrophoblast
completions of meiosis II? T
A. Degeneration of the zona pellucida E. Extraembryonic visceral mesoderm,
B. Sperm penetration of the corona radiate cytotrophoblast, and syncytiotrophoblast
C. Formation of a female pronucleus T
D. Appearance of the blastocyst 375. A 16-year-old girl presents on May 10 in
E. Completion of cleavage obvious emotional distress. On questioning,
she relates that on May 1 she experienced
370. Which of the following components plays sexual intercourse for the first time, without
the most active role in invading the using any means of birth control. Most of her
endometrium during blastocyst anxiety stems from her fear of pregnancy.
implantation? What should the physician do to alleviate her
A. Epiblast fear?
B. Syncytiotrophoblast T A. Prescribe diazepam and wait to see if she
C. Hypoblast misses her next menstrual period
D. Extraembryonic somatic mesoderm B. Use ultrasonography to document pregnancy
E. Extraembryonic visceral mesoderm C. Order a laboratory assay for serum HCG
T
371. Between which two layers is the D. Order a laboratory assay for serum
extraembryonic mesoderm located? progesterone
A. Epiblast and hypoblast E. Prescribe diethylstilbestrol (“morningafter
B. Syncytiotrophoblast and cytotrophoblast pill”)
C. Syncytiotrophoblast and endometrium
376. Carcinoembryonic antigen (CEA) is an E. Angiogenesis
oncofetal antigen that is generally associated
with which one of the following tumors? 382. The first indication of gastrulation in the
A. Hepatoma embryo is
B. Germ cell tumor A. formation of the primitive streak T
C. Squamous cell carcinoma B. formation of the notochord
D. Colorectal carcinoma T C. formation of the neural tube
E. Teratocarcinoma D. formation of extraembryonic mesoderm
E. formation of tertiary chorionic villi
377. A 42-year-old woman presents with
complaints of severe headaches, blurred 383. Somites may differentiate into which of
vision, slurred speech, and loss of muscle the following?
coordination. Her last pregnancy 5 years ago A. Urogenital ridge
resulted in a hydatidiform mole. Laboratory B. Kidneys
results show a high hCG level. Which of the C. Notochord
following conditions is a probable diagnosis? D. Epimeric and hypomeric muscles T
A. Vasa previa E. Epithelial lining of the gastrointestinal tract
B. Placenta previa
C. Succenturiate placenta 384. Intermediate mesoderm will give rise to
D. Choriocarcinoma T the
E. Membranous placenta A. Neural tube
B. Heart
378. At what location does the amniotic cavity C. Kidneys and gonads T
develop? D. Somites
A. Between the cytotrophoblast and E. Notochord
syncytiotrophoblast
B. Within the extraembryonic mesoderm 385. The developing embryo has a distinct
C. Between the endoderm and mesoderm human appearance by the end of
D. Within the hypoblast A. Week 4
E. Within the epiblast T B. Week 5
C. Week 6
379. At the end of week 2 of development (day D. Week 7
14), what is the composition of the embryonic E. Week 8 T
disk?
A. Epiblast only 386. The lateral mesoderm is divided into two
B. Epiblast and hypoblast T distinct layers by the formation of the
C. Ectoderm and endoderm A. Extraembryonic coelom
D. Ectoderm, mesoderm, and endoderm B. Intraembryonic coelom T
E. Epiblast, mesoderm, and hypoblast C. Cardiogenic region
D. Notochord
380. Which germ layers are present at the end E. Yolk sac
of week 3 of development (day 21)?
A. Epiblast only 387. Very often the first indication a woman
B. Epiblast and hypoblast has that she is pregnant is a missed menstrual
C. Ectoderm and endoderm period. In which week of embryonic
D. Ectoderm, mesoderm, and endoderm T development will a woman experience her
E. Epiblast, mesoderm, and hypoblast first missed menstrual period?
A. Start of week 3 T
381. Which process establishes the three B. Start of week 4
definitive germ layers? C. Start of week 5
A. Neurulation D. Start of week 8
B. Gastrulation T E. End of week 8
C. Craniocaudal folding
D. Lateral folding
388. A female newborn was found to have a 393. Which structure is derived from the same
large midline tumor in the lower sacral area, embryonic primordium as the dorsal root
which was diagnosed as a sacrococcygeal ganglia?
tumor. Which of the following courses of A. Gonads
treatment is recommended for this child? B. Kidney
A. Immediate chemotherapy and radiation C. Pineal gland
treatment D. Liver
B. Surgical removal by age 6 months T E. Adrenal medulla T
C. Surgical removal at age 4–5 years
D. Surgical removal at age 13–15 years 394. Which structure is derived from the same
E. No treatment because this tumor normally embryonic primordium as the kidney?
regresses with age A. Gonads T
B. Epidermis
389. A woman has her pregnancy suddenly C. Pineal gland
terminated due to intrauterine fetal death. At D. Liver
autopsy, the fetus shows severe pallor, E. Adrenal medulla
generalized edema, and hepatosplenomegaly.
Which of the following would you suspect? 395. During the later stages of pregnancy,
A. VATER maternal blood is separated from fetal blood
B. β-Thalassemia minor by the
C. β-Thalassemia major A. Syncytiotrophoblast only
D. Hydrops fetalis T B. Cytotrophoblast only
E. VACTERL C. Syncytiotrophoblast and cytotrophoblast
D. Syncytiotrophoblast and fetal endothelium
390. The specialized group of mesenchymal T
cells that aggregate to form blood islands E. Cytotrophoblast and fetal endothelium
centrally and primitive blood vessels
peripherally are called 396. The maternal and fetal components of the
A. Fibroblasts placenta are
B. Cardiac progenitor cells T A. Decidua basalis and secondary chorionic
C. Angioblasts villi
D. Myoblasts B. Decidua capsularis and secondary chorionic
E. Osteoblasts villi
C. Decidua parietalis and tertiary chorionic villi
391. The epiblast is capable of forming which D. Decidua capsularis and villous chorion
of the following germ layers? E. Decidua basalis and villous chorion T
A. Ectoderm only
B. Ectoderm and mesoderm only 397. The intervillous space of the placenta
C. Ectoderm and endoderm only contains
D. Ectoderm, mesoderm, endoderm T A. Maternal blood T
E. Mesoderm and endoderm only B. Fetal blood
C. Maternal and fetal blood
392. A male newborn has a hemangioma on D. Amniotic fluid
the left frontotemporal region of his face and E. Maternal blood and amniotic fluid
scalp. The cells forming the hemangioma are
derived from which of the following cell 398. A young insulin-dependent diabetic
layers? woman in her first pregnancy is concerned
A. Ectoderm only that her daily injection of insulin will cause a
B. Mesoderm only T congenital malformation in her baby. What
C. Endoderm only should the physician tell her?
D. Ectoderm and mesoderm A. Insulin is highly teratogenic; discontinue
E. Endoderm and mesoderm treatment
B. Insulin does not cross the placental
membrane T
C. Insulin crosses the placental membrane but 403. A 19-year-old woman in week 32 of a
is degraded rapidly complication- free pregnancy is rushed to the
D. Insulin will benefit her baby by increasing emergency department because of profuse
glucose metabolism vaginal bleeding. The bleeding subsides, but
E. Insulin crosses the placental membrane but afterward no fetal heart sounds can be heard,
is not teratogenic indicating intrauterine fetal death. The
woman goes into labor and delivers a
399. What is a normal amount of amniotic stillborn infant. On examination of the
fluid at term? afterbirth, a velamentous placenta is detected.
A. 50 ml Although not much can be done at this point,
B. 500 ml what is the diagnosis?
C. 1000 ml T A. Placenta previa
D. 1500 ml B. Vasa previa T
E. 2000 ml C. Hydatidiform mole
D. Premature rupture of the amniochorionic
400. Which of the following does not pass membrane
through the primitive umbilical ring? E. Amniotic band syndrome
A. Allantois
B. Amnion T 404. A 32-year-old pregnant woman at 30
C. Yolk sac weeks of gestation comes to her physician
D. Connecting stalk because of excess weight gain in a 2-week
E. Space connecting the intraembryonic and period. Ultrasonography reveals
extraembryonic coeloms polyhydramnios. Which fetal abnormality is
most likely responsible for the
401. Which of the following best describes the polyhydramnios?
placental components of dizygotic twins? A. Bilateral kidney agenesis
A. One placenta, two amniotic sacs, one B. Umbilical cord knots
chorion C. Velamentous placenta
B. One placenta, two amniotic sacs, two D. Hypoplastic lungs
chorions E. Esophageal atresia T
C. Two placentas, two amniotic sacs, one
chorion 405. A 25-year-old pregnant woman at 17
D. Two placentas, two amniotic sacs, two weeks of gestation comes to her OB/GYN for
chorions T a normal examination. During routine blood
E. One placenta, two amniotic sacs, two tests, her serum α-fetoprotein (AFP)
chorions concentration is found to be markedly
decreased for her gestational age. Which
402. A 26-year-old pregnant woman abnormality will the physician need to rule
experiences repeated episodes of bright red out based on these low AFP levels?
vaginal bleeding at week 28, week 32, and A. Spina bifida
week 34 of pregnancy. The bleeding B. Anencephaly
spontaneously subsided each time. Use of C. Omphalocele
ultrasound shows that the placenta is located D. Down syndrome T
in the lower right portion of the uterus over E. Esophageal atresia
the internal os. What is the diagnosis?
A. Hydatidiform mole 406. Which of the following arteries gives rise
B. Vasa previa to the axis artery of the lower limb?
C. Placenta previa T A. External iliac artery
D. Placental abruption B. Femoral artery
E. Premature rupture of the amniochorionic C. Profunda femoris artery
membrane D. Umbilical artery T
E. Inferior gluteal artery
407. The femur develops from which of the 413. On physical examination, a 35-year old
following? man was found unable to stand from stooping
A. Somite mesoderm position. Which nerve is suspected to be
B. Lateral plate mesoderm T damaged:
C. Intermediate mesoderm A. Superior gluteal nerve
D. Extraembryonic mesoderm B. Inferior gluteal nerve
E. Sclerotome mesoderm C. Sciatic nerve
D. Obturator nerve
408. The rectus femoris muscle develops from E. Femoral nerve
which of the following?
A. Posterior condensation T 414. An ultrasound examination reveals an
B. Anterior condensation occlusion of femoral artery at the proximal
C. Lateral plate mesoderm portion of the adductor canal. Which of the
D. Extraembryonic mesoderm following arteries, in this case, will most likely
E. Sclerotome mesoderm provide the collateral circulation to the thigh:
A. Descending branch of lateral circumflex
409. Which of the following muscles will the femoral
posterior divisions of the lumbosacral plexus B. Descending branch of medial circumflex
innervate? femoral
A. Semitendinosus C. Descending genicular
B. Semimembranosus D. Popliteal
C. Long head of biceps femoris E. First perforating branch of profunda femoris
D. Rectus femoris T
E. Gastrocnemius 415. An MRI examination reveals that the
popliteal vessels were injured when distal
410. During weeks 6–8, the lower limb bud fragment of the fractured femur was pulled
will rotate posteriorly. Which of the following muscle is
A. Medially 90° T responsible in displacing the distal fractured
B. Laterally 90° fragment:
C. Medially 180° A. Soleus
D. Laterally 180° B. Gastrocomius
E. No rotation occurs C. Semitendinosus
D. Semimembranosus
411. A CT scan examination reveals a very E. Gracilis
large cyst in the popliteal fossa, compressing
the tibial nerve. Which of the following 416. A 50 years old man slipped, now
movements will most likely be affected: complaint of inability to flex his leg at the
A. Dorsiflexion of the foot knee joint and extend his thigh at the hip
B. Planter flexion of the foot joint. Which of the following muscle was
C. Flexion of the thigh affected as a revealed of this accident:
D. Inversion of the foot A. Semitendinosus
E. Eversion of the foot B. Sartorius
C. Gracilis
412. A patient is involved in a motorcycle D. Quadriceps femoris
wreck that resulted in avulsion of the skin E. Adductor magnus
over the anterolateral aspect of the leg and
ankle. Which of the following structure is 417. An examination of your patient’s injured
most likely destroyed with this type of injury: knee reveals excessive posterior movement of
A. Anterior tibial nerve the tibia on the femur. The chief ligament
B. Extensor digitorum longus muscle preventing posterior sliding of the tibia on the
C. Dorsalis pedis artery femur is:
D. Superficial peroneal nerve A. Fibular collateral ligament
E. Deep peroneal nerve B. Tibial collateral ligament
C. Oblique popliteal ligament
D. Anterior cruciate ligament 422. A 21-year old man was involved in a
E. Posterior cruciate ligament motorcycle accident, resulting in destruction
of the groove on the plantar surface of the
418. A young boy complains of numbness of cuboid bone. Which of the following muscle
dorsum of his right foot and inability to tendon is most likely crushed:
dorsiflex and evert the foot. Which is the A. Flexor hallucius longus
most probable site of the nerve compression B. Peroneus brevis
that resulted in these symptoms: C. Peroneus longus
A. Popliteal fossa D. Tibialis anterior
B. Neck of the fibula E. Tibialis posterior
C. Lateral compartment of the leg
D. Anterior compartment of the leg 423. Radiograph examination of patient
E. Posterior compartment of the leg reveals avulsion fracture of greater
trochanter. Which of the following muscles
419. A patient visits the outpatient department would still continue to function normally:
with an infection in the first digital cleft of A. Obturator internus
foot. On examination the physician found B. Obturator externus
following lymph node group to be enlarged: C. Gluteus medius
A. Vertical group of superficial inguinal lymph D. Gluteus maximus
nodes E. Gluteus minimus
B. Horizontal group of superficial inguinal
lymph nodes 424. A 39-year old woman was found to have a
C. Deep inguinal lymph nodes hernial sac in the right femoral triangle with
D. Popliteal lymph node a marked weakness in adduction at the right
E. Lymph node of cloquet hip joint. Which nerve was likely compressed
by this herniation:
420. Arteriography of an old female reveals a A. Femoral
possible cause for her limb pain. The artery B. Inferior gluteal
that was occluded is one that passes above the C. Obturator
superior border of the interosseus--- D. Popliteal
membrane. Which of the following arteries is E. Sciatic
most likely affected:
A. Profunda femoris 425. After refractive force, a direct stab
B. Popliteal wound in the upper inner quadrant of the
C. Posterior tibial thigh, buttock the patient his walking very
D. Peroneal difficult. Her left hip seemed down to be tilted
E. Anterior tibial the left joint off the ground. Which nerve is
possibly damaged:
421. A 50 years old passenger received a A. Femoral
wound to upper medial thigh, died of B. Inferior gluteal
bleeding in less than 5 minutes. What was the C. Obturator
most likely nature of his injury: D. Peroneal
A. Femoral artery was cut at the inguinal E. Superior gluteal
ligament
B. Femoral artery was cut at the apex of the 426. Which of the following transport induces
femoral triangle conformational change in transmembrane
C. Femoral artery was cut at the hiatus magnus proteins:
D. Femoral vein was transected A. Simple diffusion
E. Profunda femoris artery was divided at its B. Active transport
origin C. Facilitated diffusion
D. Osmosis
E. All of the above
427. The adjacent peripheral doublets in cilia are 432. Which of the following describes the number
joined by highly extensible proteins called: of chromosomes and amount of DNA in a
A. Radial gamete:
B. Dyosine A. 46 chromosome 1N
C. Necis B. 46 chromosome 2N
D. Ascememe C. 23 chromosome 1N
E. Desceme D. 23 chromosome 2N
E. 23 chromosome 4N
428. Which statement best describes the gap
junctions: 433. The initial morphological changes
A. It extends as a zone around the apical perpendicular to gastrulation is:
perimeter of adjacent cells A. Formation of the prechordal plate
B. It possesses dense plagues composed of B. Migration of laterally placed ectoderm cells
desmoplakins towards midline
C. It permits the passage of ---- from one cell to C. Formation of the primitive streak
an adjacent cell D. Formation of the notochordal pre
D. Its adhesion is dependent upon calcium ions E. Determination of the epithelial line the
E. It possesses transmembrane linker hype—
glycoproteins
434. Intermediate mesoderm is responsible for
429. The reversible process in which one type of formation of:
epithelial tissue undergoes transformation A. Somites
into another type is called: B. Bone and muscles
A. Anaplasia C. Excretory action of the urimary system
B. Metaplasia D. Dermis and mucocutaneous tissue of the
C. Neoplasia E. None of the above
D. Dysplasia
E. None of the above 435. The ectoderm overlying the notochord:
A. Becomes thickened to form the neural plate
430. Synovial fluids examination of a patient with B. Develops into the epidermis
rheumatoid arthritis reveals the presence of C. Is a direct communication with the yolk sac
type-II collagen. Which of the following cavity
tissues is eroded in this case: D. Forms the spinal cord and somites
A. Vascular endothelium E. None of the above
B. Compact bone
C. Vascular smooth muscle 436. A young patient has stab wound at the
D. Articular cartilage mid inguinal region. Which structure is most
E. Synovial membrane likely to be injured:
A. Femoral vein
431. During meiosis-I, pairing of homologous B. Femoral nerve
chromosomes occur, which permits large C. Femoral artery
segments of DNA to be exchanged. What is D. Popliteal artery
this process called: E. Inguinal ligament
A. Synapsis
B. Nondisjunction 437. While injecting the intramuscular
C. Slignment injection into the gluteal region. Doctor must
D. Crossing over be careful about which of the following
E. Disjunction structure:
A. Femoral nerve
B. Sciatic nerve
C. Saphenous nerve
D. Lateral cutaneous nerve of thigh
E. Ilioinguinal nerve
438. Young lady came with swelling below the 443. Which muscle of anterior compartment of
inguinal ligament which was reducible. On thigh producing flexion at one joint and
examination it is below and lateral to pubic extension at second joint:
tubercle is most likely: A. Sartorius
A. Femoral hernia B. Vastus medialis
B. Inguinal hernia C. Pectineus
C. Inguinal L-N D. Vastus lateralis
D. Psoas abscess E. Psoas major
E. Femoral artery aneurysm
444. A young boy is playing football in ground
439. Lady with the compressing swelling in the with his friends, during hitting the football he
adductor canal has compress all of the fall down and felt severe pain, he was brought
following structures except: to hospital. On X-ray examination there was
A. Femoral artery torn of sacrotuberous ligament was
B. Femoral vein diagnosed. Which of the following structure
C. Saphenous nerve are crossing from greater sciatic foramen and
D. Nerve to vastus medialis lesser sciatic foramen:
E. Nerve to vastus lateralis A. Sciatic nerve
B. Superior gluteal artery
440. While observing the cell under electron C. Inferior gluteal artery
microscope, the histologist saw an organelle D. Pudendal vessels
having curved steaks, membrane sacs, small E. Posterior cutaneous nerve of thigh
vesicles and large vacuoles. He named it as:
A. Mitochondria 445. A man was hit by a motorcycle on his left
B. Golgi complex hip. After the accident, when he stands on left
C. Smooth endoplasmic reticulum foot, he falls on right side. Which of the
D. Lysosomes following muscles is involved:
E. Centrosomes A. Left gluteus maximus
B. Left gluteus medius
441. Which of the following epithelium is not C. Left gluteus minimus
capable to adjust its shape in response to any D. Left gluteus medius and minimus
influence that changes the area of epithelial E. Left gluteus maximus, medius and minimus
surface:
A. Simple cuboidal epithelium 446. 16-year old male received a superficial cut
B. Pseudo stratified epithelium on the lateral side of his foot. Four days later
C. Stratified squamous epithelium the patient has high fever and swollen lymph
D. Stratified cuboidal epithelium nodes. Which group of nodes will first receive
E. Transitional epithelium lymph from the infected wound:
A. Popliteal
442. A woman felt a small swelling at the top B. Vertical group of superficial inguinal
of her thigh that was diagnosed as the C. Deep inguinal
femoral hernia. Femoral hernia passes D. Horizontal group of superficial inguinal
through which of the following structure: E. Internal iliac
A. Retroinguinal space
B. Femoral canal 447. A malignant cancer (melanoma) of the
C. Femoral sheath skin covering the buttocks (gluteal area) is
D. Femoral ring spread via lymphatics to the:
E. Henter’s canal A. Horizontal group of superficial lymph nodes
B. Vertical group of superficial lymph nodes
C. Popliteal nodes
D. Internal iliac nodes
E. External iliac nodes
448. A general surgeon was showing popliteal 453. As elderly lady had a fall in bathroom,
artery in the popliteal fossa to the after which she was unable to get up and
postgraduate students. At what level, this walk. On X-ray examination it was found to
artery divides into anterior tibial and be fracture of neck of femur. There is a
posterior tibial arteries: danger of avascular necrosis of head of
A. Lower borders of gastroconemius femur. The integrity of which artery is
B. Upper border of biceps femoris essential for prevention of this condition:
C. Lower border of popliteus muscle A. Lateral circumflex femoral
D. Within the adductor canal B. Medial circumflex femoral
E. Before entering into the adductor canal C. Obturator
D. Inferior gluteal
449. A 45-years old lady with uncontrolled E. Superior gluteal
diabetes after missing her menstrual cycle is
worried about her baby may not born with 454. During a repair of a wound, the collagen
any birth defect. Which of the following fibers actively synthesized by:
period is potentially sensitive regarding A. Macrophages
development and defects: B. Fibrocytes
A. 1st – 2nd week C. Fibroblast
B. 3rd – 4th week D. Cells with pigment granules
C. 4th – 5th week E. Mast cells
D. 3rd – 8th week
E. 4th – 10th week 455. An elderly man visited the doctor with
complains of cramp like condition after
450. A 20 years old girl visited to her lady walking for 10 minutes. After examination
doctor with irregular menstrual cycle and Doppler ultrasound a diagnosis of
diagnosed of an ovulation was made. At Intermittent claudication was made. The
which stage of cell division ovum is arrested: ultrasound revealed the blockage of right
A. Metaphase of meiosis-II femoral artery near its lower end. The blood
B. Prophase of meiosis-I supply of right lower leg is maintained to
C. Metaphase of meiosis-I prevent gangrene. Which of the following
D. Prophase of meiosis-II arteries will not take part in collateral
E. Telophase of meiosis-I circulation:
A. Muscular branches of femoral artery
451. After injury and severe bleeding from B. Genicular branches of femoral artery
foot area, the doctors decided to ligate the C. Perforating branches of profunda femoral
femoral artery in adductor canal. The artery
surgeon is not worried about damage to: D. Muscular branches of popliteal artery
A. Nerve to vastus medialis E. Genicular branches of popliteal artery
B. Femoral vein
C. Femoral nerve 456. A boy of 10 years of age developed
D. Saphenous nerve infection of lateral border of little toe of foot.
E. Terminal part of obturator nerve The infection from this site can be first
transmitted through lymphatics to which of
452. In embryo the first system to starts the following groups of lymph nodes:
developing is nervous system. It begins with A. Vertical chain of superficial inguinal group
the process of neuralation which is B. Horizontal chain of superficial inguinal
characterized by: group
A. Spindle formation C. Deep inguinal group
B. Formation of notochord D. Popliteal group
C. Formation of neural tube E. External iliac group
D. Formation of primitive streak
E. Formation of extraembryonic mesoderm
457. A normal pregnancy is maintained by the 462. In foot injuries, if person is not able to
action of trophoblast on corpus luteum. The flex the metatarsophalangeal joints and
corpus luteum will continue to function for extend the interphalangeal joints. Structure
which of the following period after likely to be affected is:
pregnancy: A. Lumbricals
A. Throughout pregnancy B. Tendon of tibialis posterior
B. 2-3 months C. Tendon of flexor digitorum longus
C. 14 days D. Flexor digitorum accessories muscle
D. 3-4 months E. Flexor digitorum brevis
E. 1-2 months
463. A begger was standing at bus stop and
458. A female of 30 years of age complained of approach the person for some tip, while
swelling in the region of upper medial side of receiving the tip the position of the hand of
thigh. The swelling was soft, non-pulsatile the bagger is in:
and reducible on lying down. There was no A. Extension
involvement of lower limb, abdomen or B. Flexion
perineal region. This kind of swelling is likely C. Supination
to be: D. Pronation
A. Enlarged inguinal lymph nodes E. Adduction
B. Psoas abscess
C. Femoral hernia 464. After accident a young male developed a
D. Saphenous varix fracture of pelvis with dislocation of hip joint.
E. Aneurysm of femoral artery The diagnosis of injury to sciatic nerve of
gluteal region is made. To confirm the
459. While examination of shoulder joint, diagnosis of sciatic nerve injury which of the
doctor asked a patient to move a joint in all following signs is not considered:
directions. The combination of all movements A. Paralysis of hamstrings
performed on shoulder joint is known as: B. All the muscles below knee are paralyzed
A. Flexion and extension C. Loss of sensation on posterior surface of
B. Flexion, abduction, extension and adduction thigh
C. Flexion, extension, lateral and medial D. Loss of sensations over greater part of ---
rotation and foot
D. Cirumduction E. Foot drop
E. Protrusion and rotrusion
465. A medical student was asked to palpate
460. Which of the following is not a the arteries in lower limb whose pulsations
characteristic of Hematoxylin-Eosin staining: can easily be felt. Which of the following
A. Nuclei appears as blue arteries cannot be palpable:
B. Cytoplasm appears as pink or red A. Femoral
C. Elastic fibers appears as blue or black B. Peroneal
D. Collagen appears as pink C. Posterior tibial
E. Muscle fiber stain as pink D. Anterior tibial
E. Dorsalis pedis
461. After injury to foot and on X-ray,
fracture was found to be in cuboid bone, 466. A young boy in an accident while hitting a
there was no sensory loss in any part of foot. bike to a hard large stone developed a
There was some difficulty in eversion of foot. fracture of distal third of femur. On
The structure likely to be damaged is: examination there is less blood flow to leg and
A. Tendon of tibialis posterior foot. The distal third of femur is rotated
B. Planter aponeurosis backward which has put a pressure on which
C. Tendon of peroneus longus of the following arteries:
D. Tendon of peroneus brevis A. Posterior tibial
E. Medial planter nerve B. Anterior tibial
C. Popliteal
D. Peroneal B. Disintigation of zona pellucida T
E. Terminal part of femoral C. Formation of decidua
D. Separation of trophoblastic cells and the
467. While examining blood supply of foot, the inner cell mass
sole of the foot receives blood supply from E. Start of cleavage
medial and lateral planter arteries along with
a supply from which of the following arteries: 472. While dissecting the human body, the
A. Dorsalis pedis artery teacher told the students about many planes
B. Femoral artery of the body. He dissected an organ vertically
C. Peroneal artery into two right and left equal halves. The plane
D. Anterior tibial artery of the cut organ is said to be:
E. Posterior tibial artery A. Oblique
B. Transverse
468. While doing an ultrasound of a mother C. Coronal
with 25th week of pregnancy, the report came D. Saggital T
with less amount of amniotic fluid. Which of E. Lateral
the following facts is not related to amniotic
fluid: 473. While examining a histological slide of a
A. Can be taken out from mother for bone. Which of the following cells will not be
examination seen in this slide:
B. Restrict the movements of fetus A. Osteophyte
C. Allows the growth of the fetus B. Fibroblast
D. Act as a barrier to infection C. Osteoblast
E. At full term amount is about 1000 ml D. Osteocyte
E. Osteoclast
469. On ultrasound examination, a pregnant
mother showed ectopic pregnancy. She was 474. A male of 60 years of age with complaints
immediately sent to gynaecologist for further of pain in the knee and difficulty in
assessment. The gynaecologist is worried movement arrived at hospital for his
about her problem. Ectopic pregnancy may examination. The doctor diagnosed this as a
be fatal to the mother because of: case of arthritis. The structure damaged is
A. Toxemia of pregnancy articular cartilage which has:
B. Destruction of vital organ due to pressure A. Has rich blood supply
exerted by the growing B. Give attachment to synovial membrane
C. Hemorrhage C. Hyaline variety
D. Occlusion of vessels in the vicinity of the D. Fibrocartilage variety
implantation site E. Elastic variety
E. Perforation of the gut
475. A woman during second trimester of
470. Cells that secretes mucus directly and pregnancy is admitted in hospital for
specifically is known as: observation of fetal distress. On ultrasound
A. Epithelial cells examination there seems to be a problem
B. Endocrine cells within the placental circulation. Which of the
C. Goblet cells following layers will not be included in the
D. Holocrine cells separation of the maternal blood from fetal
E. Exocrine cells blood:
A. Syncytiotrophoblast
471. A woman of child bearing age complaints B. Cytotrophoblast
of repeated abortions. The attending doctor is C. Amniotic membrane
worried about the implantation of ovum in D. Chorionic mesoderm
the uterus of mother. Most important event E. Endothelium of blood vessel
resulting in implantation of fertilized ovum
is:
A. Formation of blastocyst
476. A new born baby is brought to hospital 480. While looking at the slide of a hyaline
with edema over sacral region showing some cartilage. The space surrounding the
tuff of hairs. The baby was diagnosed as a chondrocytes is known as:
case of sacro-coccygeal teratoma. This A. Canaliculi
condition is due to: B. Intercellular space
A. Excessive production of intraembryonic C. Vacuole
mesoderm D. Lacuna
B. Absence of notochord E. Basilateral space
C. Retention of tissue of primitive streak
D. Less amount of amniotic fluid 481. While examining a slide of an organ
E. Non rupture of oropharyngeal membrane (trachea) from respiratory system. The
epithelium is identified is:
477. A young man while walking briskly on A. Transitional
uneven surface put its ankle under over B. Stratified squamous
eversion with lateral rotation. He was not C. Pseudo-stratified ciliated columnar
able to put his weight on affected ankle. D. Simple squamous
Which was swollen on lateral side. There was E. Simple cuboidal
no apparent fracture seen on radiograph. A
diagnosis of injury to medial collateral 482. During a ward round, a group of students
ligament was made. This ligament is not check the patellar (knee jerk) reflex. The
attached to: reflex determines the integrity of which
A. Medial malleolus segment of spinal cord:
B. Navicular bone A. L4, L5
C. Calcaneus B. L5, S1
D. Talus C. S1, S2
E. Medial cuneiform D. S2, S3
E. L2, L3, L4
478. A long distance runner who wears joggers
for long time is suffering from pain on the 483. A young female meet with an accident
medial side of foot and heel. The sensations whom she got a deep cut across her thigh just
are abnormal. The nerve likely to be below inguinal ligament. Femoral nerve had
compressed is: been severed through deep cut. Injury to
A. Superficial peroneal femur/femoral nerve below inguinal ligament
B. Medial planter will not show:
C. Lateral planter A. Failure of quadriceps to contract
D. Tibial B. Loss of sensations over anterior and medial
E. Sural parts of thigh
C. Loss of sensation to medial border of foot
479. While studying the muscles and its D. Paralysis of iliacus muscle
supply, the concept of motor unit is that: E. Complete loss of full extension at knee on
A. It is a supporting cell for motor neuron walking
B. It is motor axon present in spinal cord
C. It is multiple muscle fibers innervated by 484. While studying the anatomy of a joint, a
single motor neuron joint with movements on only one axis and
D. It is free nerve ending around muscle fibers showing one set of movements is known as
E. It is a single muscle fiber innervated by hinge joint. The typical example of this type
single motor neuron of joint is:
A. Wrist joint
B. Shoulder joint
C. Elbow joint T
D. Median atlanto-axial joint
E. Sternoclavicular joint
485. A fast bowler in cricket while running patient was diagnosed as having an ectopic
towards wicket to bowl, suddenly started pregnancy. Which of the following organs is
limping and unable to walk properly with most likely to provide a normal site of
severe pain in attempting to walk. He has fertilization:
injured a tendon attached to ischial A. Fundus of the uterus
tuberosity. The group of muscle affected in B. Ampulla of the uterine tube
this case is: C. Fimbriae
A. Hamstrings D. Infundibulum of the uterine tube
B. Quadriceps femorus E. Body of the uterus
C. Adductors of thigh
D. Abductors of thigh 491. The connective tissue covering which
E. Psoas major and iliacus envelops the whole muscle is known as:
A. Epimysium
486. An oocyte is developing normally, at B. Endomysium
which of the time does the second meiotic C. Perimysium
division of oocyte will be completed: D. Sarcolemma
A. At fertilization E. Aponeurosis
B. During ovulation
C. In the morula 492. A boy was playing football game. He
D. In the blastocyst kicks the ball into net and makes a goal.
E. Upon implantation Which of the following movement did he
perform at hip joint:
487. A medical student is asked for the cells of A. Flexion
nervous system, which are non-excitable, B. Extension
abundant and cause the myelination inside C. Adduction
the central nervous system. These cells are: D. Abduction
A. Schwann cells E. Circumduction
B. Oligodendrocytes
C. Microglial cells 493. While studying the anatomy of a joint, a
D. Astrocytes joint with movements on only one axis and
E. Ependymal cells showing one set of movements is known as
hinge joint. The typical example of this type
488. The long bones are generally confined to: of joint is:
A. Thorax A. Wrist joint
B. Head and neck B. Shoulder joint
C. Appendicular skeleton C. Elbow joint T
D. Vertebral column D. Median atlanto-axial joint
E. Axial skeleton E. Sternoclavicular joint
489. On examination of synovial joint, doctor 494. According to the arrangement of fibers
asked a patient to move a joint in all and line of pull, following muscle is a good
directions. The combination of all movements example of multipennate muscle:
performed on shoulder joint is known as: A. Deltoid
A. Flexion and extension B. Pectineus
B. Flexion, abduction, extension and adduction C. Rectus femoris
C. Flexion, extension, lateral and medial D. Sartorius
rotation E. Semitendinosus
D. Circumduction
E. Protrusion and retraction 495. A histologist was examining the slide
under microscope, the tissue cells showed
490. A 26 years old woman has an increased basophilia, the increase basophilia
amenorrhea, followed by uterine bleeding, is due to:
pelvic pain and pelvic mass. Her obstetrician A. Microtubules
performed a thorough examination, and the B. Mitochondria
C. Golgi body J. First perforating
D. Rough endoplasmic reticulum
E. Smooth endoplasmic reticulum 501. A patient presents with a thrombosis in
the popliteal vein. This thrombosis most
496. A weak and anemic child was advised for likely causes reduction of blood flow in which
bone marrow biopsy. Which of the following of the following veins?
bone is suitable for this procedure: F. Greater saphenous
A. Femur G. Lesser saphenous
B. Humerus H. Femoral
C. Iliac joint I. Posterior tibial
D. Rib J. Anterior tibial
E. Tibia
502. A 27-year-old patient exhibits a loss of
497. Cells that secretes mucus directly and skin sensation and paralysis of muscles on the
specifically is known as: plantar aspect of the medial side of the foot.
A. Epithelial cells Which of the following nerves is most likely
B. Endocrine cells damaged?
C. Goblet cells F. Common peroneal
D. Holocrine cells G. Tibial
E. Exocrine cells H. Superficial peroneal
I. Deep peroneal
498. A thoracic surgeon is going to collect a J. Sural
portion of the great saphenous vein for
coronary bypass surgery. He has observed 503. A 7 years old boy having history of high
that this vein runs: grade fever and fits, advised by his doctor for
A. Posterior to the medial malleolus lumbar puncture. Which one of the following
B. Into the popliteal fossa anatomical structure will be used as
C. Anterior to the medial condyles of the tibia landmark for lumbar puncture:
and femur A. Femur
D. Superficial to the fascia lata of the thigh B. Iliac crest
E. Along with the femoral artery C. Ischium
D. Pubis
499. A 47-year-old woman is unable to invert E. Vertebrae
her foot after she stumbled on her driveway.
Which of the following nerves are most likely 504. A construction worker falls feet first from
injured? a roof. He sustains a fracture of the groove on
F. Superficial and deep peroneal the undersurface of the sustentaculum tali of
G. Deep peroneal and tibial the calcaneus bone. Which of the following
H. Superfi cial peroneal and tibial muscle tendons is most likely torns:
I. Medial and lateral plantar A. Flexor digitorum brevis
J. Obturator and tibial B. Flexor digitorum longus
C. Flexor hallucis brevis
500. A 62-year-old woman slips and falls on D. Flexor hallucis longus
the bathroom floor. As a result, she has a E. Tibialis posterior
posterior dislocation of the hip joint and a
fracture of the neck of the femur. The woman 505. A patient experiences paralysis of the
undergoes hip surgery. If all of the arteries muscle that originate from the femur and
that are part of the cruciate anastomosis of contributes directly to the stability of the knee
the upper thigh are ligated, which of the joint. Which of the following muscles is
following arteries maintains blood flow? involved:
F. Medial femoral circumflex A. Vastus lateralis
G. Lateral femoral circumflex B. Semimembranosus
H. Superior gluteal C. Sartorius
I. Inferior gluteal D. Biceps femoris (long head)
E. Rectus femoris
511. A young boy was brought to the ward
506. A patient is unable to prevent anterior after sharp knife injury. On examination
displacement of the femur on the tibia when there was loss of sensation from lower lateral
the knee is flexed. Which of the following part of leg upto lateral side of little toe. This
ligaments is most likely damaged: shows the damage to following nerve:
A. Anterior cruciate A. Common peroneal
B. Fibular collateral B. Deep peroneal
C. Patellar C. Saphenous
D. Posterior cruciate D. Superficial peroneal
E. Tibial collateral E. Sural
507. An electrician is screwing a hole with 512. The most commonly used stain in
screw driver to make an electrical point in the histology is the:
wall. Which of the following movement he is A. Hematoxyline and eosin
performing: B. Hematoxyline and picric acid
A. Flexion of arm C. Orange and toufidine blue
B. Extension of arm D. Silver staining
C. Pronation
D. Supination 513. A person is suffering from demyelination
E. Pronation and supination of the peripheral nervous system. Which of
the following cells would be affected:
508. The lymphatics are characterized by all of A. Astrocytes
the following except: B. Ependymal cells
A. No valves C. Microglial cells
B. No distinct layers D. Schwann cells
C. Multiple valves E. Oligodendocytes
D. Lined by simple endothelium
E. Freely communicating with veins 514. Epidermis determines the colour of skin
due to which of the following factor:
509. A 22 years old man presented to his A. Thickness
family physician with a laceration of the B. Blood flow
fibrous sheets and bands that cover his body C. Keratin
under the skin and invest the muscles. Which D. Melanin
of the following structures would most likely E. Lymphatics
be injured:
A. Tendon 515. Which type of collagen is present in the
B. Fascia dermis:
C. Synovial tendon sheath A. Collagen type-I
D. Aponeurosis B. Collagen type-II
E. Ligament C. Collagen type-III
D. Collagen type-IV
510. A 46 years old male patient with high E. Collagen type-V
blood pressure was examined in the
emergency department, and his physician 516. A young lady went to a doctor for skin
found a leakage of blood from the blood problem on her face, where doctor told her
vessels that normally carries richly that she is suffering from acne disease. The
oxygenated blood. Which of the following cause of this problem is due to defect of
vessels would most likely be damaged: following structure of skin:
A. Superior vena cava A. Epidermis
B. Pulmonary arteries B. Dermis
C. Pulmonary veins C. Hairs
D. Portal vein D. Sebaceous glands
E. Femoral vein E. Sweat glands
B. Peroneus longus
517. A patient with chest pain comes to doctor, C. Peroneus brevis
who after ECG advises him for angiography. D. EHL
Which vessel of lower limb is commonly used E. EDL
for this procedure:
A. Femoral artery 523. The foot drop is commonly associated
B. Femoral vein with paralysis of the:
C. Posterior tibial artery A. Tibial nerve
D. Popliteal artery B. Common peroneal nerve
E. Popliteal vein C. Femoral nerve
D. Medial plantar nerve
518. A 27 years old patient exhibits a loss of E. Lateral plantar nerve
skin sensation and paralysis of muscles on the
plantar aspect of the medial side of the foot. 524. The tendon passes beneath the
Which of the following nerves is most likely sustentaculum tali in the sole of the foot is:
damaged: A. Tibialis posterior tendon
A. Common peroneal B. Peroneus longus
B. Tibial C. Peroneus brevis
C. Superficial peroneal D. FDL
D. Deep peroneal E. FHL
E. Sural
525. The key stone of lateral longitudinal arch
519. Fracture of the neck of the femur results is:
in avascular necrosis of the femoral head, A. Calcaneus
probably resulting from the lack of blood B. Head of the talus
supply from which of the following arteries: C. Navicular
A. Obturator D. Cuboid
B. Superior gluteal E. 4th and 5th metatarsals
C. Inferior gluteal
D. Medial femoral circumflex 526. A boy playing soccer has suffered trauma
E. Lateral femoral circumflex to the medial meniscus from a blow to the
lateral aspect of the knee. The knee is
520. A 30 years old married man went to unstable, what other structure is most likely
doctor with complaint of having no child. The to be injured:
doctor advised him for analysis of semen. The A. Deltoid ligament
normal count of spermatozoa in semen per B. Lateral meniscus
ejaculation is: C. Posterior cruciate ligament
A. 200-400 thousands D. Tibial / medial collateral ligament
B. 200-400 millions E. Fibular / lateral collateral ligament
C. 20-40 millions
D. 50-100 millions 527. The tendon of which muscle with synovial
E. 2-4 millions sheath passes through both superior +
inferior extensor retinacula:
521. Intercellular junctions show the A. Tibialis anterior
junctional complex in the: B. EHL
A. Connective tissue C. EDL
B. Epithelial tissue D. Peroneus tertius
C. Muscular tissue E. EDB
D. Nervous tissue
E. Neuroglial tissue 528. The action of lumbricals is to:
A. Flexes the distal phalanges of the lateral four
522. The muscle which performs both actions toes
of dorsiflexion and inversion is: B. Abduct the big toe
A. Peroneus tertius C. Flexes the big toe
D. Extends the toes at the interphalangeal joints E. Basal cells are columnar in stratified
E. Extends the toes at the metatarsophalangeal epithelium
joints
535. The type of sebaceous gland is:
529. The ligament occupied by sinus tarsi is: A. Apocrine
A. Medial talocalcaneal ligament B. Holocrine
B. Lateral talocalcaneal ligament C. Merocrine
C. Plantar calcaneonavicular ligament D. Endocrine
D. Bifurcated ligament E. Eccrine
E. Interosseous talocalcaneal ligament
536. Which structure is responsible for the
530. A woman wearing high heels has fallen linkage of the intermediate filaments of cells
and twisted / sprained her ankle from to the basement membrane:
excessive inversion. Most likely which A. Macula adherens
structure will be torned: B. Zonula adherens
A. The deltoid ligament C. Hemidesmosomes
B. The lateral ligament D. Zonula occludens
C. The tendocalcaneus E. Focal contacts
D. The interosseous ligament
E. The peroneal retinaculum 537. A group of students are discussing about
the different types of tissue, but they are more
531. Ovulation occurs during: interested to discuss about the labile tissue
A. The 1st week of uterine cycle which is:
B. Last week of uterine cycle A. Epithelium
C. Middle of uterine cycle B. Muscular tissue
D. Menstrual phase of uterine cycle C. Nervous tissue
E. Just before the menstrual phase D. Connective tissue
E. Blood
532. The normal allocation of chromosomes in
a sperm is: 538. A boy aged 15 years suffering from
A. 23 autosomes paraplegia (damage of both lower limb) due
B. 23 autosomes + a sex chromosome to road accident. Which of the following
C. 22 autosomes + a sex chromosome structure is involved for such deformity:
D. 22 paired autosomes + a pair of sex A. Injury to the artery
chromosome B. Injury to the nerve
E. 22 autosomes C. Injury to the vein
D. Injury to the lymphatic
533. At birth all of the following gametes in the E. Injury to the bone
ovary are:
A. Haploid 539. A boy aged 20 years having swelling in
B. Oogonia feet. Which one of the following structure is
C. Primary oocytes responsible for such edema (swelling):
D. Haploid oocytes A. Arterial obstruction
E. Graffian follicles B. Venous obstruction
C. Damage of the nerve
534. Features of epithelia: D. Muscular hypertrophy
A. Transitional epithelium lines the urinary E. Fracture of bone
tract
B. Cells in the pseudo stratified epithelium are 540. A clinical condition is characterized by
in contact with extracellular matrix the demyelination of peripheral nervous
C. Mitosis occurs in all layers of stratified system. Name the commonly affected cells:
squamous epithelium A. Oligodendrocyte
D. Simple cuboidal epithelium lines the large B. Ependymal cells
excretory ducts C. Astrocyte
D. Schwann cells C. Articular surfaces are covered by hyaline
E. Neuroglial cells cartilage
D. Intervertebral disc is elastic type of cartilage
541. Cell bodies of preganglionic autonomic E. Cartilages are highly vascular
neurons for sympathetic system are located
in: 547. A boy aged 18 years, running for bowling.
A. Spinal cord (gray matter) Which common movement at knee joint he is
B. Ventral horn going to perform:
C. Paravertebral sympathetic ganglia A. Flexion + adduction
D. Lateral horns from T1 L2 B. Flexion + extension
E. Brain C. Flexion + abduction
D. Extension + medial rotation
542. A muscle which initiates particular E. Extension + lateral rotation
movement and remains active constantly is
called: 548. A school boy was punished by teacher to
A. Agonist do sit and stand (up + down) for 20 times.
B. Synergistic Which possible movement at knee joint he
C. Fixator will perform:
D. Antagonist A. Abduction + adduction
E. None B. Abduction + medial rotation
C. Adduction + lateral rotation
543. A highly vascularized anatomical D. Flexion + extension
structure supplied by two named arteries. E. Extension + abduction
Which will be the type of anastomosis:
A. Actual anastomosis 549. A young lady is eating burger in
B. Physiological anastomosis restaurant by using her both hands. Which
C. Potential anastomosis possible movement at shoulder joint she is
D. End arterial anastomosis going to perform for both hands:
E. Anatomical anastomosis A. Extension
B. Flexion
544. A lady aged more than 60 years having C. Lateral rotation
carcinoma (cancer) of breast. What is the D. Medial rotation
probable route of spread of carcinoma: E. Abduction + adduction
A. Venous route
B. Arterial route 550. Electrician was busy to tight screw on
C. Lymphatic route wall. Which probable movement is performed
D. Capillary route by him:
E. Arteriovenous route A. Flexion of arm
B. Extension of elbow
545. A young lady went to dermatologist (skin C. Supination + pronation
specialist) for the treatment of ACNE. Which D. Adduction + abduction
of the following statement regarding acne is E. Flexion + extension of elbow
true:
A. It is due to infection of sweat glands 551. A boy aged 20 years playing football
B. It is due to infection of lymphatic system match, he threw a ball into the net and made
C. It is due to infection of sebaceous gland a goal. Which possible movement at hip joint
D. It is due to infection of venous system he did performed:
E. It is due to infection of epidermis A. Extension
B. Medial rotation
546. Regarding cartilage, which one of the C. Lateral rotation
following is true: D. Flexion
A. Perichondrium is insensitive E. Circumduction
B. Cartilages are rich in lymphatics
552. A mother is holding a baby by folding C. Deep external pudendal vein
both arms. Which possible movement at D. Superficial external pudendal vein
shoulder region she is going to perform: E. Accessory vein
A. Adduction + abduction
B. Adduction + flexion 558. Which of the following is not the action of
C. Flexion + extension Sartorius:
D. Medial rotation of both arm A. Flexion of thigh
E. Lateral rotation of both arm B. Abduction of thigh
C. Lateral rotation of thigh
553. Lady demonstrating her students in D. Adduction of thigh
exercise club about various movements at E. Medial rotation of leg
multiple axis. Which variety of synovial joint
is responsible to perform such movements: 559. Straight head of rectus femoris takes
A. Hinge variety origin from:
B. Ellipsoid variety A. Ilium above the acetabulum
C. Ball and socket variety B. Anterior inferior iliac spine
D. Pivot variety C. Anterior superior iliac spine
E. Plane variety D. Base of greater trochanter
E. Superior ramus of pubis
554. A teacher was discussing about shaft of
femur during class. Which statement 560. The femoral ring is bounded by the
regarding shaft is true: following structures, except:
A. Avascular part of bone A. Inguinal ligament
B. Containing perichondrium B. Superior ramus of pubis
C. Covered by hyaline cartilage C. Lacunar ligament
D. Compact bone D. Femoral vein
E. Devoid of nerve supply E. Femoral artery
555. Medical student is asked for a cells of 561. Which of the following muscle is flexor of
nervous system, which are non-excitable, thigh:
abundant and cause myelination of central A. Adductor longus
nervous system. Such cell is one of the B. Vastus lateralis
following: C. Gracilis
A. Neuron D. Psoas major
B. Astrocyte E. Obturator externus
C. Microglia 562. A femoral hernia descends through the
D. Oligodendrocyte femoral canal and the neck of sac lies:
E. Ependymal cells A. Below and lateral to pubic tubercle
B. Above and medial to the pubic tubercle
556. A young lady of 25 years old wearing high C. At the saphenous opening
heels, suddenly got severe pain because of D. In the obturator canal
ankle sprain when she was going fast to the E. Lateral to the iliacus muscle
car parking. Which anatomical structure is
supposed to be involved: 563. The following structures pass through the
A. Ligament subsartorial (adductor) canal, except:
B. Joint A. Branch of obturator nerve
C. Muscle B. Nerve to vastus lateralis
D. Tendon C. The femoral artery
E. Nerve D. The saphenous nerve
E. The femoral vein
557. All of the following are tributaries of
great saphenous vein, except: 564. The floor of femoral triangle is formed by
A. Superficial epigastric vein following muscles, except:
B. Superficial circumflex iliac vein A. Pectineus
B. The adductor longus 571. Regarding medial surface of tibia:
C. The iliacus A. Is mostly subcutaneous
D. The psoas B. Gives origin to tibialis anterior
E. The adductor brevis C. Gives insertion to gracilis
D. Gives origin to Sartorius, semimembranosus
565. All of the following are branches of and semitendonosus
femoral nerve, except: E. Gives attachment to interosseous ligament
A. Medial cutaneous nerve of thigh
B. Intermediate cutaneous nerve of thigh 572. The extensor hallucis longus:
C. Saphenous nerve A. Takes origin from talus
D. Muscular branch to pectineus B. Is inserted into the base of distal phalanx of
E. Muscular branch to obturator externus big toe
C. Is supplied by superficial peroneal nerve
566. All of the following are parts of femur, D. The tendon passes behind the inferior
except: extensor retinaculum
A. Greater trochanter E. Is attached to distal phalanx of second toe
B. Linea aspera
C. Iliac tubercle 573. Housemaid knee is inflammation of:
D. Intercondylar notch A. Popliteal bursa
E. Neck B. Supra-patellar bursa
C. Pre-patellar bursa
567. Patella is an example of: D. Superficial infra-patellar bursa
A. Long bone E. Deep infra-patellar bursa
B. Short bone
C. Irregular bone 574. 2nd, 3rd and 4th dorsal metatarsal arteries
D. Flat bone are branches of:
E. Sesamoid bone A. Lateral tarsal artery
B. Medial tarsal artery
568. The deep fascia: C. First dorsal metatarsal artery
A. Provides attachment to some of the muscles D. Arcuate artery
B. Is well developed in the abdominal walls E. Dorsalis pedis artery
C. Form retinacula at the elbow and knee
D. Is insensitive 575. An 18 years old student was doing part
E. Is absent in the neck time work delivering pizzas on his
motorcycle, the student weave in and out of
569. Regarding the skin: traffic whenever there was a holdup. On one
A. Main factor determining the colour of the occasion he misjudged the gap between two
skin is its degree of blood supply vehicles and outer surface of his left knee hit
B. The number of melanocytes varies in car bumper. On examination in the
different races emergency room, he was found to have
C. Thin skin has great concentration of sweat extensive paralysis of muscles of anterior and
glands lateral compartment of left leg. There was
D. Skin of palm and sole has great evidence of diminished sensation down the
concentration of sebaceous glands anterior and lateral sides of leg and dorsum
E. Base of hair follicle is attached to the of foot and toes including the medial side of
arrector pilli muscle big toe. The structure damaged is:
A. Inferior transverse ligament
570. Which one of the following muscle is not B. Anterior tibial artery
found in the superficial fascia: C. Superficial peroneal nerve
A. Dartos muscle of scrotum D. .
B. Muscles of facial expression E. .
C. Platysma
D. Palmaris brevis 576. Following is derived from ectoderm:
E. Arrector pilli A. Blood vessels
B. Muscles E. Arcuate artery
C. Digestive system 583. Peroneus longus is inserted at:
D. Lens of eye A. Base of fifth metatarsal bone
E. Vertebrae B. Base of first metatarsal bone and medial
cuneiform
577. Regarding collagen: C. Lateral surface of the shaft of fibula
A. Type-I is abundant in hyaline cartilage D. Base of distal phalanx of big toe
B. Type-II forms thick bundles E. Calcaneum
C. Type-III is most abundant type
D. Type-IV is major component of basement 584. Branches of which nerve supply skin
membrane beneath the nail beds:
E. Collagen fibers are easily stretched A. Superficial peroneal nerve
B. Deep peroneal nerve
578. Regarding the cartilage: C. Tibial nerve
A. Matrix of hyaline cartilage is basophilic D. Saphenous nerve
B. Fibrocartilage is the most common form of E. Sural nerve
cartilage
C. Elastic cartilage is found in intervertebral 585. The foot is inverted and dorsiflexed by:
discs A. Peroneus tertius
D. Articular cartilage has perichondrium B. Tibialis anterior
E. Cartilage matrix have lacunae containing C. External hallucis longus
osteocytes D. External digitorum longus
E. Tibialis posterior
579. Brown adipose tissue differs from white
adipose tissue in which of the following ways: 586. The following statements concerning the
A. Exports of fatty acids dorsalis pedis artery are correct, except:
B. Role as a thermal insulator A. It is the continuation of the anterior tibial
C. Use of fatty acids to produce heat artery
D. Activation of adenylyl cyclase system B. It enters the sole of the foot by passing
E. Initiation of shivering through the 1st intermetatarsal space
C. It can be palpated on the foot between the
580. All of the following structures pass tendons of tibialis anterior and external
beneath the extensor retinacula from medial hallucis longus
to lateral, except: D. It joins the lateral planter artery
A. Tendon to tibialis anterior E. On its lateral side lies the terminal part of
B. Deep peroneal nerve deep peroneal nerve
C. Anterior tibial or dorsalis pedis arteries
D. Small saphenous vein 587. All of the following are features of foot
E. Tendon of peroneus tertius drop (injury to common peroneal nerve).
Except:
581. Which of the following muscle dorsiflexes A. Muscles of anterior compartment of the leg
the foot at the ankle joint: are paralyzed
A. Peroneus longus B. It is commonly injured in fractures of the
B. Extensor digitorum brevis neck of fibula
C. Extensor hallucis brevis C. There is loss of sensation on lateral side of
D. Tibialis posterior leg
E. Tibialis anterior D. There is loss of sensation on the medial
border of foot
582. All of the following are the branches of E. There is loss of sensation on the dorsum of
dorsalis pedis artery, except: the foot and toes
A. Lateral tarsal artery
B. 1st dorsal metatarsal artery 588. All of the following are anterior bursae of
C. 2nd dorsal metatarsal artery the knee joint, except:
D. Medial tarsal artery A. Supra-patellar bursa
B. Pre-patellar bursa A. Is bipennate
C. Popliteal bursa B. Partly arises from interosseous membrane
D. Superficial infra-patellar bursa C. Is attached to many tarsal bones
E. Deep infra-patellar bursa D. Everts the foot
E. Helps in plantar flexion of the foot
589. Cruciate ligaments of the knee joint are
supplied by: 595. The flexor digitorum longus receives the
A. Medial superior genicular artery attachments from all of the following, except:
B. Lateral superior genicular artery A. The flexor hallucis longus
C. Middle genicular artery B. The flexor accessorius
D. Medial inferior genicular artery C. The unipennate lumbricals
E. Lateral inferior genicular artery D. The bipennate lumbricals
E. The interossei muscles
590. A motorcycle accident results in
destruction of the groove in the lower surface 596. Which of the following muscles is able to
of the cuboid bone. Which one of the dorsiflex and invert the foot:
following muscle tendon is most likely A. Extensor digitorum longus
damaged: B. Tibialis anterior
A. Flexor hallucis longus C. Tibialis posterior
B. Peroneus brevis D. Peroneus longus
C. Peroneus longus E. Peroneus brevis
D. Tibialis anterior
E. Tibialis posterior 597. Regarding the muscles of the sole:
A. Flexor digitorum brevis is the muscle of 1st
591. All about the gastrocnemius are true, B. Flexor digitorum accessorius is inserted into
except: flexor hallucis longus
A. Forms the lower boundary of the popliteal C. All lumbricals are bipennate
fossa D. Tendon of peroneus longus forms the 3rd last
B. Inserts by way of the tendocalcaneus E. All interossei are unipennate
C. Plantar flexes the foot at the ankle joint
D. Is surrounded by a synovial sheath near its 598. Regarding interossei muscles:
insertion A. The plantar interossei are thicker than dorsal
E. Is supplied by the tibial nerve interossei
B. The plantar interossei arise by two heads
592. The soleus muscle: C. The dorsal interossei are four in number
A. Is the most superficial muscle in the calf D. The plantar interossei abduct the toes
B. Has the tibial vessels and nerve lying E. The dorsal interossei adduct the toes
between it and the gastrocnemius muscle
C. Is attached superiorly by its medial head to 599. All are true regarding the muscles of the
the medial femoral condyle sole, except:
D. Is attached inferiorly by the tendocalcaneus A. The belly of flexor hallucis brevis splits into
to the posterior surface of the calcaneus --- parts
E. Is supplied by the common peroneal nerve B. Lumbricals maintain the extension of the
interphalangeal joints
593. All are true about popliteus, except: C. The commonest site of seasmoid bone is the
A. Unlocks the knee joint --- of extensor hallucis longus
B. Get its nerve supply through femoral nerve D. The lumbricals arise from the tendons of the
C. Tendon of origin is intracapsular digitorum longus
D. Laterally rotates the femur on a fixed tibia E. The interossei arise from the metatarsals
E. Attached to the pit below lateral epicondyle
of femur 600. Regarding the anatomical position:
A. Face and palms of the hands are directed
594. All are true about tibialis posterior, forward
except: B. The person is standing erect
C. The upper limbs by the sides C. End arteries do not anastomose with their
D. A+B+C neighbours
E. None of the above D. The true capillaries have smooth muscle
cells
601. What is true regarding the skin: E. Sinusoids have narrow and regular lumen
A. It represents 60% of the total body weight
B. The dermis is the most outer layer of the 607. What is true about nervous system:
skin A. There are 31 pairs of spinal nerves
C. Epidermis is continuously shed and replaced B. All spinal nerves are mixed nerves
D. Sweat gland is a small, sack shaped gland C. Cell bodies of motor neurons lie in the
E. Sebaceous gland is an elongated tubular anterior horn of the gray matter
structure D. Cell bodies of primary sensory neurons lie
in the posterior root ganglion
602. What is true about muscle: E. All of the above
A. Insertion is the end of the muscle that
remains fixed during its contraction 608. All of the following statements are true,
B. Tendon is its fleshy contractile part except:
C. Actin filaments are thick A. Coronal plane divides the body into front
D. Myosin filaments are thin and back halves
E. None of the above B. Abduction is the movement of the limb
away from the midline
603. What is true about bone: C. Keratinocytes are cells that make melanin
A. Is a dead tissue D. Arrector pilli is a smooth muscle
B. Femur is an example of a modified long E. Apocrine glands are more developed in
bone women
C. Clavicle is a miniature long bone
D. Patella is a sesamoid bone 609. All the following statements are true,
E. All of the above except:
A. Each smooth muscle fiber anastomoses with
604. Regarding the joints: the neighbouring fibers at intercalated discs
A. Fibrous joints permits a great degree of B. Oblique fasciculi make the muscle more
movement powerful
B. Wrist joint is hinge type C. Motor point is the site where the motor
C. Gomphosis is a type of cartilaginous joint nerve enters the nerve
D. Costochondral junction is an example of D. Bone marrow manufactures the blood cells
secondary cartilaginous joint E. Pneumatic bones act as air conditioning
E. Pubic symphysis is an example of secondary chambers for the inspired air
cartilaginous joint